ChaseDream

标题: 【每日逻辑练习第二季】【1-8】 [打印本页]

作者: bananazoo    时间: 2011-11-26 23:37
标题: 【每日逻辑练习第二季】【1-8】
今天的练习来了~
大家快乐的做题吧~
【精练】
1.People who have specialized knowledge about a
scientific or technical issue are systematically
excluded from juries for trials where the issue is
relevant. Thus, trial by jury is not a fair means of
settling disputes involving such issues.
Which one of the following, if true, most seriously
weakens the argument?
Weaken Question Problem Set  NO.9
(A) The more complicated the issue being litigated,
the less likely it is that a juror without
specialized knowledge of the field involved
will be able to comprehend the testimony
being given.
(B) The more a juror knows about a particular
scientific or technical issue involved in a
trial, the more likely it is that the juror will be
prejudiced in favor of one of the litigating
parties before the trial begins.
(C) Appointing an impartial arbitrator is not a fair
means of settling disputes involving
scientific or technical issues, because
arbitrators tend to favor settlements in which
both parties compromise on the issues.
(D) Experts who give testimony on scientific or
technical issues tend to hedge their
conclusions by discussing the possibility of
error.
(E) Expert witnesses in specialized fields often
command fees that are so high that many
people involved in litigation cannot afford
their services.

【逻辑链】
29.
To reduce the danger to life and property posed by major earthquakes, scientists have been investigating several techniques for giving advance warning of dangerous earthquakes.  Since catfish swim erratically before earthquakes, some investigators have proposed monitoring catfish to predict dangerous earthquakes.
Which of the following, if true, most seriously undermines the usefulness of the proposal?
(A) In Japan, which is subject to frequent earthquakes, the behavior of catfish has long been associated with earthquakes.
(B) Mechanical methods for detecting earthquakes have not proved effective.
(C) Tremors lead to the release of hydrogen sulfide gas into water, thereby causing various fish and shellfish to behave erratically.
(D) Careful construction can reduce the dangers posed by earthquakes.
(E) Even very slight, fleeting tremors cause catfish to swim erratically.


30.
A city plans to attract new citizens with new housing and new facilities such as parks, recreation centers, and libraries.  One component of the city's plan is to require that developers seeking permission to build this new housing provide these additional facilities at no cost to the city.
Which of the following, if true, would point to a possible flaw in the city's plan?
(A) Developers would pass along their costs to the buyer, thereby raising the cost of housing units beyond the ability of likely purchasers to afford them.
(B) Light, nonpolluting industries have located in the area, offering more jobs and better-paying jobs than do the more-established industries in the area.
(C) Other towns and cities nearby have yet to embark on any comparable plans to attract new citizens.
(D) Most developers see the extra expense of providing municipal facilities as simply one of the many costs of doing business.
(E) Studies show that purchasers of new houses, especially first-time buyers, rank recreational resources as an important factor in deciding to buy a particular house.


31.
Manufacturers issue cents-off coupons to get consumers to try their brand of product with the hope that the consumers who try their brand will switch their brand loyalty.  So in the initial marketing of their new brand X, Hartman Industries should issue cents-off coupons, thereby attracting a large segment of potential consumers as loyal customers.
Which of the following, if true, casts the most serious doubt on the likelihood that the marketing strategy recommended above will have the result that is claimed?
(A) Many consumers are unlikely to try new brands of products unless offered an inducement to do so.
(B) The consumers whose purchases are strongly influenced by cents-off coupons tend not to become loyal customers of any particular brand.
(C) Many grocery stores attract customers by doubling the face value of manufacturer's coupons.
(D) Typically less than one-third of the coupons issued by a manufacturer are redeemed by consumers.
(E) A marketing campaign that uses cents-off coupons is most effective when combined with a television advertising campaign.


32.
State spokesperson:  Many businesspeople who have not been to our state believe that we have an inadequate road system.  Those people are mistaken, as is obvious from the fact that in each of the past six years, our state has spent more money per mile on road improvements than any other state.
Which of the following, if true, most seriously undermines the reasoning in the spokesperson's argument?
(A) In the spokesperson's state, spending on road improvements has been increasing more slowly over the past six years than it has in several other states.
(B) Adequacy of a state's road system is generally less important to a businessperson considering doing business there than is the availability of qualified employees.
(C) Over the past six years, numerous businesses have left the spokesperson's state, but about as many businesses have moved into the state.
(D) In general, the number of miles of road in a state's road system depends on both the area and the population of the state.
(E) Only states with seriously inadequate road systems need to spend large amounts of money on road improvements.

作者: bananazoo    时间: 2011-11-26 23:39
逻辑链答案:  E A B E
精炼解析:
Question #3. December 2003 LSAT, Section #1, #20. The correct answer choice is (B)
The first sentence is a premise, and the second sentence is the conclusion of this argument. To attack this
conclusion, look for an answer choice that shows that the exclusion of knowledgeable individuals from
scientific or technical issue trials is a fair way of proceeding in these trials.
Answer choice (A): This is an Opposite answer that strengthens the conclusion. If specialized knowledge
of these issues makes it more likely that the juror can comprehend the testimony being given, then these
individuals should not be excluded from juries, and their exclusion makes trial by jury an unfair means of
resolving a dispute.
Answer choice (B): This is the correct answer. If the specialized knowledge is likely to produce a
prejudice in a juror, then by all means they should be excluded from the jury. Thus, instead of trial by jury
being an unfair means, it is made more fair by the exclusion of these individuals. The answer is a tricky
one because most people initially think the answer agrees with the argument. It agrees with the principle of
the premise, but not with the conclusion drawn from that premise.
Answer choice (C): This answer simply notes that arbitrators are not a fair means of settling scientific or
technical issue debates. This has no impact on the fairness of jury trials involving these same issues.
Answer choice (D): This answer is about the experts testifying at scientific or technical issue trials. This
information does not attack the claim that jury trials are unfair because of the exclusion of jurors with
knowledge of these issues.
Answer choice (E): This answer can be eliminated by reasoning similar to that used to eliminate answer
choice (D).
——Weaken Question Problem Set  NO.3
作者: xeyyxzty    时间: 2011-11-26 23:41
1.premise:People who have specialized knowledge about a scientific or technical issue are systematically excluded from juries for trials where the issue is relevant.
conclusion: trial by jury is not a fair means of settling disputes involving such issues
呃...想不出来...
(A) The more complicated the issue being litigated, the less likely it is that a juror without specialized knowledge of the field involved will be able to comprehend the testimony being given.
--right
(B) The more a juror knows about a particular scientific or technical issue involved in a trial, the more likely it is that the juror will be prejudiced in favor of one of the litigating parties before the trial begins.
--weaken--right
(C) Appointing an impartial arbitrator is not a fair means of settling disputes involving scientific or technical issues, because arbitrators tend to favor settlements in which both parties compromise on the issues.
--an impartial arbitrator--irrelevant
(D) Experts who give testimony on scientific or technical issues tend to hedge their conclusions by discussing the possibility of error.
--experts, not members of juries--irrelevant
(E) Expert witnesses in specialized fields often command fees that are so high that many people involved in litigation cannot afford their services.

--the fees are irrelevant
反了......原来是没有认真看题...trial by jury ~not people with specialized knowledge
问题是专家被排除在外,陪审团就不公正~反驳,排除后更公正
2.background information:To reduce the danger to life and property posed by major earthquakes, scientists have been investigating several techniques for giving advance warning of dangerous earthquakes.  
premise:catfish swim erratically before earthquakes
conclusion:some investigators have proposed monitoring catfish to predict dangerous earthquakes
earthquakes-->catfish swim erratically,but you cannot get the conclusion that when the fish swim erratically, there is to be an earthquake
(A) In Japan, which is subject to frequent earthquakes, the behavior of catfish has long been associated with earthquakes.
--support
(B) Mechanical methods for detecting earthquakes have not proved effective.
-irrelevant
(C) Tremors lead to the release of hydrogen sulfide gas into water, thereby causing various fish and shellfish to behave erratically.
--irrelevant
(D) Careful construction can reduce the dangers posed by earthquakes.
--irrelevant
(E) Even very slight, fleeting tremors cause catfish to swim erratically.
--too sensitive--right
思路不对~可是想的也没错吧
3. premise:require that developers seeking permission to build this new housing provide these additional facilities at no cost to the city.
conclusion:attract new citizens with new housing and new facilities such as parks, recreation centers, and libraries.
what if people don't care about the new housing and facilities?
(A) Developers would pass along their costs to the buyer, thereby raising the cost of housing units beyond the ability of likely purchasers to afford them.
--weaken--right
(B) Light, nonpolluting industries have located in the area, offering more jobs and better-paying jobs than do the more-established industries in the area.

--irrelevant or support
(C) Other towns and cities nearby have yet to embark on any comparable plans to attract new citizens.

--irrelevant
(D) Most developers see the extra expense of providing municipal facilities as simply one of the many costs of doing business.

--irrelevant
(E) Studies show that purchasers of new houses, especially first-time buyers, rank recreational resources as an important factor in deciding to buy a particular house.


--support
思路又一次远离正确轨道...
4. background information:Manufacturers issue cents-off coupons to get consumers to try their brand of product with the hope that the consumers who try their brand will switch their brand loyalty.
premise: in the initial marketing of their new brand X, Hartman Industries should issue cents-off coupons
conclusion:attracting a large segment of potential consumers as loyal customers
没有想法...
(A) Many consumers are unlikely to try new brands of products unless offered an inducement to do so.
--support
(B) The consumers whose purchases are strongly influenced by cents-off coupons tend not to become loyal customers of any particular brand.

--weaken the proposal--right
(C) Many grocery stores attract customers by doubling the face value of manufacturer's coupons.

--irrelevant
(D) Typically less than one-third of the coupons issued by a manufacturer are redeemed by consumers.

--irrelevant
(E) A marketing campaign that uses cents-off coupons is most effective when combined with a television advertising campaign.


--irrelevant
5.premise: in each of the past six years, our state has spent more money per mile on road improvements than any other state.
conclusion: Many business people who have not been to our state believe that we have an inadequate road system.  they are wrong.
whether the costs to build a certain length of roads are higher than other states?
(A) In the spokesperson's state, spending on road improvements has been increasing more slowly over the past six years than it has in several other states.
--???
(B) Adequacy of a state's road system is generally less important to a businessperson considering doing business there than is the availability of qualified employees.
--irrelevant
(C) Over the past six years, numerous businesses have left the spokesperson's state, but about as many businesses have moved into the state.
--irrelevant
(D) In general, the number of miles of road in a state's road system depends on both the area and the population of the state.
--irrelevant
(E) Only states with seriously inadequate road systems need to spend large amounts of money on road improvements.
--right
思路呀~~~
作者: Jane412    时间: 2011-11-26 23:54
板凳~
作者: Jane412    时间: 2011-11-26 23:54
给逻辑链~
作者: bananazoo    时间: 2011-11-27 00:00
占楼~表示熊熊你太迅速啦~
还有今天昨天耳的都留在明天做~
一会寝室要睡啦~明天有人要考公务员~加油~
作者: 小意达de花儿    时间: 2011-11-27 10:15
继续占位嘎
作者: qiuhua01234567    时间: 2011-11-27 10:19
今天比较容易,终于让我喘口气了

1 weaken 20S
Premise: .People who have specialized knowledge about a
scientific or technical issue are systematically
excluded from juries for trials where the issue is
relevant.
Conclusion: trial by jury is not a fair means of
settling disputes involving such issues
(A)    The more complicated the issue being litigated,
the less likely it is that a juror without
specialized knowledge of the field involved
will be able to comprehend the testimony
being given.
---------------------------------------------------------------strong
(B) The more a juror knows about a particular
scientific or technical issue involved in a
trial, the more likely it is that the juror will be
prejudiced in favor of one of the litigating
parties before the trial begins.
------------------------------------------------------------------correct
(C) Appointing an impartial arbitrator is not a fair
means of settling disputes involving
scientific or technical issues, because
arbitrators tend to favor settlements in which
both parties compromise on the issues.
---------------------------------------------------------------------irrelevent
(D) Experts who give testimony on scientific or
technical issues tend to hedge their
conclusions by discussing the possibility of
error.
-------------------------------------------------------------------------irrelevent
(E) Expert witnesses in specialized fields often
command fees that are so high that many
people involved in litigation cannot afford
their services.
--------------------------------------------------------------------------irrelevent
3.    weaken 20S
background: To reduce the danger to life and property posed by major earthquakes, scientists have been investigating several techniques for giving advance warning of dangerous earthquakes.
Premise: catfish swim erratically before earthquakes,
Conclusion: some investigators have proposed monitoring catfish to predict dangerous earthquakes
(A)    In Japan, which is subject to frequent earthquakes, the behavior of catfish has long been associated with earthquakes.
---------------------------------------------------------------------------------------------------------strong
(B) Mechanical methods for detecting earthquakes have not proved effective.
------------------------------------------------------------------------------------------------------------irrelevent
(C) Tremors lead to the release of hydrogen sulfide gas into water, thereby causing various fish and shellfish to behave erratically.
-----------------------------------------------------------------------------------------------------irrelevent
(D) Careful construction can reduce the dangers posed by earthquakes.
--------------------------------------------------------------------------------------------------irrelevent
(E) Even very slight, fleeting tremors cause catfish to swim erratically.
-------------------------------------------------------------------------------------------------------correct
4.    flaw 20S
ways: attract new citizens with new housing and new facilities such as parks, recreation centers, and libraries
component: developers seeking permission to build this new housing provide these additional facilities at no cost to the city
(A)    Developers would pass along their costs to the buyer, thereby raising the cost of housing units beyond the ability of likely purchasers to afford them.
-----------------------------------------------------------------------------------------------------correct
(B) Light, nonpolluting industries have located in the area, offering more jobs and better-paying jobs than do the more-established industries in the area.
----------------------------------------------------------------------------------------------------irrelevent
(C) Other towns and cities nearby have yet to embark on any comparable plans to attract new citizens.
------------------------------------------------------------------------------------------------------irrelevent
(D) Most developers see the extra expense of providing municipal facilities as simply one of the many costs of doing business.
------------------------------------------------------------------------------------------------------------- need money.wrong
(E) Studies show that purchasers of new houses, especially first-time buyers, rank recreational resources as an important factor in deciding to buy a particular house.
----------------------------------------------------------------------------------------------------------------irrelevent.
5 weaken 25S
Background: Manufacturers issue cents-off coupons to get consumers to try their brand of product with the hope that the consumers who try their brand will switch their brand loyalty
Ways: in the initial marketing of their new brand X, Hartman Industries should issue cents-off coupons
Result: attracting a large segment of potential consumers as loyal customers.
(A)    Many consumers are unlikely to try new brands of products unless offered an inducement to do so.
------------------------------------------------------------------------------------------------------------have inducement
(B) The consumers whose purchases are strongly influenced by cents-off coupons tend not to become loyal customers of any particular brand.
--------------------------------------------------------------------------------------------------------correct
(C) Many grocery stores attract customers by doubling the face value of manufacturer's coupons.
-------------------------------------------------------------------------------------------------------irrelevent
(D) Typically less than one-third of the coupons issued by a manufacturer are redeemed by consumers.
--------------------------------------------------------------------------------------------------------irrelevent
(E) A marketing campaign that uses cents-off coupons is most effective when combined with a television advertising campaign.
--------------------------------------------------------------------------------------------------------irrelevent.
6    25S weaken
Background:Many businesspeople who have not been to our state believe that we have an inadequate road system
Premise: that in each of the past six years, our state has spent more money per mile on road improvements than any other state
Conclusion: Those people are mistaken
A)    In the spokesperson's state, spending on road improvements has been increasing more slowly over the past six years than it has in several other states.
-------------------------------------------------------------------------------------------------but improve
(B) Adequacy of a state's road system is generally less important to a businessperson considering doing business there than is the availability of qualified employees.
------------------------------------------------------------------------------------------------irrelevent
(C) Over the past six years, numerous businesses have left the spokesperson's state, but about as many businesses have moved into the state.
-----------------------------------------------------------------------------------------------irrelevent
(D) In general, the number of miles of road in a state's road system depends on both the area and the population of the state.
-------------------------------------------------------------------------------------------------irrelevent
(E) Only states with seriously inadequate road systems need to spend large amounts oaf money on road improvements.
-----------------------------------------------------------------------------------------------correct.
作者: 南瓜0729    时间: 2011-11-27 10:59

作者: zz42050524    时间: 2011-11-27 21:28
精炼 Weaken  30s
P: People specialized in something are excluded from jury of trail, which has knowledge relevant.
C: It is not fair to have jury with specialized knowledge.
这个题不太懂,理解费劲 。先看答案 我选A 先去见解析
看了xeyyxzty的话才恍然大悟,题都没看看懂逻辑意思。 自己写的结论也是错的。
这道题记录再去好好理解下。

逻辑链 2. Weaken 31s
Scientists found a kind of fish swim erratically before earthquake.
Scientists decided to use the fish warning earthquake.
Pre: The fish also swim erratically in other conditions.
E, 有点偏,意思差不多就是weaken 这个fish预测的不准或者没有价值

3. flaw 33
The city wants to attract citizens with buildings parks and facilities. It requires the developer of buildings to provide facilities with no cost.
Flaw: The developer wouldn’t receive the request even abandon the project in the city, because the cost is much higher than its profits.
又偏了, 羊毛出在养身上, 商人无力不起早, 肯定转嫁给消费者。 答案A

4. Weaken 40s
P: The company wants customs who try its products will become its loyalty, it issues cents-off coupons.

Pre: Customs who try its products with the cents-off coupons will also be attracted by other coupons.
B , 猜中

5. Weaken 26s
P:  Businessmen who have not been to our state say that we are lacking road system. Last six years, we have invested more money per mile than any other state.

Since they lack road system, they put more money in it.
E 老提 第一反应就是答案, 一般来说weaken 投资的多并不代表现在拥有的道路网就足够,一个意思。
作者: fox0923    时间: 2011-11-28 03:48
你们太强劲了~~继续补作业~~
谁能帮我解释一下昨天的精炼部分?我的逻辑框架还没太搞懂~~
作者: fox0923    时间: 2011-11-28 04:17
精练-----------------32s--------------------weaken
P:People with technical knowledge are excluded from the juries for trials.
C: Thus, the trial is not fair if involved such issues.
Prephrase: The trial is not fair if the juries don't judge the trials fairly.
analysis:
(A) The more complicated the issue being litigated,
the less likely it is that a juror without
specialized knowledge of the field involved
will be able to comprehend the testimony
being given.---------------------------------more complex case, less likely the juror can handle the case. but this is not what the conclusion mainly talk about.更正思维:没想到这其实也是support。
如果要使juror的knowledge越来越多了,那么他就可以自己处理了,不需要那些people with speical technical knowledge了。

(B) The more a juror knows about a particular
scientific or technical issue involved in a
trial, the more likely it is that the juror will be
prejudiced in favor of one of the litigating
parties before the trial begins.-----------------yeah。If a juror knows about technical knowledge, then he or she might be prejudiced in favor of one party. 是不是潜意识里证明了juror会导致unfair,而并非people who know technical knowledge are excluded from the trial会导致unfairness。
阴险选项: 还是bible解释得好。
The answer is a tricky
one because most people initially think the answer agrees with the argument. (这就是我刚开始想的,后来才转过弯来)It agrees with the principle of the premise, but not with the conclusion drawn from that premise.(遇到类似的选项看到和原文提到的info相反的对象,有可能就是被argue的对象,即weaken)
(C) Appointing an impartial arbitrator is not a fair
means of settling disputes involving
scientific or technical issues, because
arbitrators tend to favor settlements in which
both parties compromise on the issues.---------arbitrator is not the point in the argument.
(D) Experts who give testimony on scientific or
technical issues tend to hedge their
conclusions by discussing the possibility of
error.--------------------------------------tend to hedge is to support the conclusion instead of weaken it.
(E) Expert witnesses in specialized fields often
command fees that are so high that many
people involved in litigation cannot afford
their services.-------------------------------this is to support the conclusion/
作者: 风无衣    时间: 2011-11-28 21:10
【精炼1-8】
weaken_55s
premise:people(good at scientific or technical issues) excluded from juries for trials systematically when the issue is relevant.
conclusion: trial by jury is not a fair means of settling disputes involving such issues.
prephrase:find a choice to states that trial by jury is a fair way of settling disputes about these issues although experts excluded from juries. There are some disadvantages when these experts participate in the trials by juries.

(A) The more complicated the issue being litigated,
the less likely it is that a juror without
specialized knowledge of the field involved
will be able to comprehend the testimony
being given.——support
(B) The more a juror knows about a particular
scientific or technical issue involved in a
trial, the more likely it is that the juror will be
prejudiced in favor of one of the litigating
parties before the trial begins.——correct
(C) Appointing an impartial arbitrator is not a fair
means of settling disputes involving
scientific or technical issues, because
arbitrators tend to favor settlements in which
both parties compromise on the issues.——irrelevant about arbitrator.
(D) Experts who give testimony on scientific or
technical issues tend to hedge their
conclusions by discussing the possibility of
error.——irrelevant about experts who give testimony.
(E) Expert witnesses in specialized fields often
command fees that are so high that many
people involved in litigation cannot afford.——irrelevant

jury 陪审团
testimony 证言,声明,证据
作者: balapupu    时间: 2011-11-28 23:29
[24s]
P: people know the scientific and technical issue is excluded from the jury if the jury is about this issue.
C: the trail is not fair.
Weaken: the people have the knowledge about the scientific and technical will can not judge the issue fair.
A:
(A) The more complicated the issue being litigated,
the less likely it is that a juror without
specialized knowledge of the field involved
will be able to comprehend the testimony
being given.-->support
(B) The more a juror knows about a particular
scientific or technical issue involved in a
trial, the more likely it is that the juror will be
prejudiced in favor of one of the litigating
parties before the trial begins.-->R
(C) Appointing an impartial arbitrator is not a fair
means of settling disputes involving
scientific or technical issues, because
arbitrators tend to favor settlements in which
both parties compromise on the issues.-->irrelevant with the arbitrators.
(D) Experts who give testimony on scientific or
technical issues tend to hedge their
conclusions by discussing the possibility of
error.-->not refute the fairness
(E) Expert witnesses in specialized fields often
command fees that are so high that many
people involved in litigation cannot afford
their services.-->irrelevant with the money

逻辑链:
1.【19s】
P:Catfish swimming erratically before the earthquake?scientists think it’s a good way to predicted the earthquake and decrease the property and life loss
weaken: the Cat fish only behave erratically just a few minutes before the earthquake, the time is too short for people get out of danger.
A:
(A) In Japan, which is subject to frequent earthquakes, the behavior of catfish has long been associated with earthquakes.-->support
(B) Mechanical methods for detecting earthquakes have not proved effective.-->support
(C) Tremors lead to the release of hydrogen sulfide gas into water, thereby causing various fish and shellfish to behave erratically.-->supprt
(D) Careful construction can reduce the dangers posed by earthquakes.-->irrelevant with the construction
(E) Even very slight, fleeting tremors cause catfish to swim erratically.-->R

2.[20s]
P: the city plan to sell buildings with entertainment facilities to attract citizens? one component of this plan the additional facilities will have no cost to the city.
Flaw: when people come to use the facility, it will bring the environment cost of the city.
A:
(A) Developers would pass along their costs to the buyer, thereby raising the cost of housing units beyond the ability of likely purchasers to afford them.-->R
(B) Light, nonpolluting industries have located in the area, offering more jobs and better-paying jobs than do the more-established industries in the area.—support
(C) Other towns and cities nearby have yet to embark on any comparable plans to attract new citizens.-->irrelevant with other cities
(D) Most developers see the extra expense of providing municipal facilities as simply one of the many costs of doing business.-->support
(E) Studies show that purchasers of new houses, especially first-time buyers, rank recreational resources as an important factor in deciding to buy a particular house.-->irrelevant with the purchase style.

3.[18s]

P: Industry implements cents off coupon to attract the loyalconsumer. -à So H Companyshould issue cents off coupon.
Weaken: this measure can not attract loyal consumer.
A:
(A) Many consumers are unlikely to try new brandsof products unless offered an inducement to do so.—>support

(B) The consumers whose purchases are stronglyinfluenced by cents-off coupons tend not to become loyal customers of anyparticular brand.-->R

(C) Many grocery stores attract customers bydoubling the face value of manufacturer's coupons.-->irrelevant with theother solution.

(D) Typically less than one-third of the couponsissued by a manufacturer are redeemed by consumers.-->irrelevant with the reorganization.

(E) A marketing campaign that uses cents-offcoupons is most effective when combined with a television advertising campaign.-->irrelevantwith the ads way.


4.做过了。


作者: ugly5552000    时间: 2011-11-29 00:41
1、Background: people who specialize in a certain issue are often excluded from juries for trials of the issue.
Premise: the trial by jury is not faire.
Prephrase: without the professionals, the jury’s horizon won’t be limited.

B
A: the argument is about whether the trial is faire, not whether the jurors are able to comprehend the testimony.
B: with a specialized juror in the jury will make the trial prejudiced, it attacks the argument.
C: no arbitrator, irrelevant
D: irrelevant
E: irrelevant
2、Background: catfish swim erratically before earthquakes.
Premise: the dangerous earthquakes can be predicted by monitoring catfish.
Prephrase: if there is a situation in addition to earthquakes that makes catfish swim erratically, the argument will be weakened.
E
A: associated with earthquakes does not mean it is effective.
B: irrelevant
C: why catfishes behave erratically is irrelevant
D: irrelevant
E: this is another circumstance which makes catfishes swim erratically, it attacks the argument.
3、Background: a city plans to attract new citizens by constructing new facilities. The new housing provide the new facilities must be free.
Prephrase: the developers could rent out the housing to get the capital for building.

A
A: it is an effective way to carry out the plan.
B: irrelevant
C: irrelevant
D: irrelevant
E: irrelevant
4、Background: manufactures issue cents-off coupons to attract consumers to switch their brand loyalty.
Premise: Hartman Industries issues cents-off coupons to attract consumers as loyal cosumers.
Prephase: if the strategy works, other manufacturers will adopt that too, then the loyal consumers will be lost soon.

B
A: the aim of manufacturers is make consumers loyal, not only to try their new brands
B: it attacks the argument
C: the strategy is irrelevant
D: irrelevant
E: irrelevant
5、Background: it is a perception that the country’s road system is inadequate.
Premise: in each of the past six years, the state has spent more money per mile on road improvements than any other state.
Prephrase: the more money was spent on improvement of the roads’ quality, not the total  mileage of the roads.
E
A: comparison with other countries is irrelevant
B: irrelevant
C: irrelevant
D: irrelevant
E: it points out the state’s problem and attacks the argument.
作者: baseboss    时间: 2012-5-21 21:27
1.Excluded the people which has special acknowledge or technical relevant from X is not a good way.peo
presume:
1.people has the revelant information about the trial will not fare to judge in lie of the whole social.

A) The more complicated the issue being litigated,
the less likely it is that a juror without
specialized knowledge of the field involved
will be able to comprehend the testimony
being given.
-->support
(B) The more a juror knows about a particular
scientific or technical issue involved in a
trial, the more likely it is that the juror will be
prejudiced in favor of one of the litigating
parties before the trial begins.
-->weaken   right
(C) Appointing an impartial arbitrator is not a fair
means of settling disputes involving
scientific or technical issues, because
arbitrators tend to favor settlements in which
both parties compromise on the issues.
-->irrelevant
(D) Experts who give testimony on scientific or
technical issues tend to hedge their
conclusions by discussing the possibility of
error.
-->irrelevant
(E) Expert witnesses in specialized fields often
command fees that are so high that many
people involved in litigation cannot afford
their services.
-->irrelevant


2.Use catfish to advanced warning earthquake.
presume:
1.how was the relationship between cstfish behavior and earthquake.
C

3.A city want to attack civic people by new housing and ralated facilities.They want the housing developers take the cost of related facilities by free.
presume:
1.No developers will do.
A

4.Manufacturies will use coupons to let customer try their brand and hope them to swith to their brand loyalty.So x need to do so at the initial.
Presume:
1.If customer swith to the brand loyalty,it is ussless to use coupons to attack customer.
B

5.State spend more money than other states to improve the road.
presume:
more money do not means good road situation.
Road situation can not be judged by the money spend.
E
作者: 卖红薯a    时间: 2012-5-22 23:12
精练
1.
1)计时:25s
2)逻辑链Situation
Background information: people who have specialized knowledge about scientific/technical issues are out of the jury for trials if the issue is relevant.
Conclusion: trial by jury isn’t a fair means of settling arguments involving such issues.
3)推测: trial by jury is a fair means < ------ people in jury who deal with the issue is not scientific/technical professional (this is fair)
4)选项分析:选A
A. Right.
B. Irrelevant.
C. Opposite.
D. Irrelevant.
E. Irrelevant. about fee

【逻辑链】
29.
1)计时:30s
2)逻辑链Situation
Background information: purpose: reduce the danger of life/property caused by earthquakes, scientists investigate techniques for giving advance warning
Premise: C swim erratically before earthquake
Conclusion: they proposed monitoring C to predict dangerous earthquake
3)推测: it’s not sure how long before the earthquake C swim erratically, 1 hour? 1 month? 1 year?
we don’t know if C usually swim regularly
4)选项分析:选E
A. Support.
B. Support.
C. Irrelevant. other fishes
D. Irrelevant. main point is prediction not reduce the dangers posed by earthquake
E. Right.

30.
1)计时:25s
2)逻辑链Situation
Background information: A city plans to attract new citizens with new housing/facilities.
Premise: it requires that developers seek permission that to build the facilities cost nothing to the city
3)选项分析:选A
A. Right.
B. Irrelevant.
C. Support.
D. Support.
E. Support.

31.
1)计时:60s
2)逻辑链Situation
Background information: producers issue cents-off coupons to get consumers to try their brand of products, hoping the consumers will switch their brand loyalty
Premise: HI should issue cents-off coupons in the initial marketing of their new brand X
Conclusion: so that they can attract many potential consumers as loyal customers
3)推测: X is a new brand, issue cents-off coupons may not attract many loyalty.
it can attract consumers but may not switch to their brand loyalty
4)选项分析:选B
A. Support the conclusion. Cents-off coupon is an inducement
B. Right.
C. Irrelevant. no new brand involved
D. Irrelevant.
E. Support the conclusion.

32.
1)计时:28s
2)逻辑链Situation
Background information: many people who haven’t been to our state think that we have an inadequate road system.
Premise: those people are wrong
Conclusion: we has spent more money on road improvements than any other state in past 6 years
3)推测: our road system may be extremely bad than other states before, even we improved it, and it’s still worse than other states.
4)选项分析:选E
A. Irrelevant. increasing speed
B. Irrelevant. which is important for a businessman
C. Irrelevant.
D. Irrelevant. the length of the road
E. Right.
作者: Threesu    时间: 2012-5-28 21:46
P: people in juries of a trial do not have special relevant knowledge
C:the trial is unfair
(A) support
(B) weaken
(C) weaken对象错误,所以应该是无关
(D) irrelevant对象错误
(E) irrelevan

B:scientists are doing something to reduce the danger of life and prosper by earthquake
Pne kind of fish swims erratically before earthquake
C:by monitoring the fish
(A)support
(B) irrelevant
C) weaken
(D) irrelevant
(E) support

错了,C是解释地震引发鱼群动的原因

B: a city wanna attract citizens by offering new faculties
P: the building are going to be proposed by people
C:no cost to the city
(A) weaken
(B) irrelevant
(C) irrelevant
(D) Most developers see the extra expense of providing municipal facilities as simply one of the many costs of doing business.
(E) irrelevant

B:companies issue coupons to attract customers and wanna make them loyal ones
P:C is offering a new brand to the market
C:issue
(A) support
(B) weaken
(C) irrelevant
(D) little weaken
(E) irrelevant

P:A spends more money on road building per mile than any other countries
C:it is wrong that A in need of more roads
(A) /
(B) /
(C) /
(D) /
(E) Only states with seriously inadequate road systems need to spend large amounts of money on road improvements.
作者: 二楼往下掉    时间: 2012-6-2 13:28
周末,终于有时间白天吧题做了。。。不用大半夜的再来发上来了。。。
第一道题完全没看懂,那个jury 和trail不理解啊。。。
废话完了。。。贴作业
1.
Trial 试验,审判
Jury 陪审团
Specialized 专业的
(1)45s weaken
(2)逻辑链
Primes:
People who have specialized knowledge about a scientific or technical issue are systematically excluded from juries for trials where the issue is relevant.
Conclusion:
Trial by jury is not a fair means of settling disputes involving such issues.
(3)example
4)A:support,说明了专家的重要性,loser
B:有专业知识的专家会在试验前有偏见,contender
C:arbitrator跟文中啥有关系?
D:专家会通过提出其中可能存在的错误来阻碍证词做出结论,这应该是什么态度的?专家=/=陪审团
E:无关选项,argument在讨论这种选择方式是不是合理

2.
(1)45s weaken
(2)逻辑链
Background:
To reduce the danger to life and property posed by major earthquakes, scientists have been investigating several techniques for giving advance warning of dangerous earthquakes.
Primes:
Catfish swim erratically before earthquakes
conclusion:
Some investigators have proposed monitoring catfish to predict dangerous earthquakes
(3)<1>方法不可行:监测不到
<2>方法不能达到目的:不在earthquake之前catfish也会swim erratically
(4)A:support,证明这个方法可行,loser
B:没提到plan-goal,loser
C:support,证明方法有效,loser
D:与plan-goal无关,loser
E:方法不可行,同<2>,weaken,contender

3.
(1)45s flaw
(2)逻辑链
Background:
A city plans to attract new citizens with new housing and new facilities such as parks, recreation centers, and libraries.
Plan:
Require that developers seeking permission to build this new housing provide these additional facilities at no cost to the city.
(4)A:开发商会把成本转嫁到消费者头上,这样无形当中增加了city的成本,contender
B:无关,loser
C:无关比较,跟别的cities没啥关系,loser
D:说明plan很好,loser
E:无关选项,与消费者对于设施周边的要求无关,loser
所以,选A

4.
(1)45s weaken
(2)逻辑链
Background:
Manufacturers issue cents-off coupons to get consumers to try their brand of product with the hope that the consumers who try their brand will switch their brand loyalty.
Plan:
new brand X——issue cents-off coupons to attract a large segment of potential consumers as loyal customers
(3)<1>不是所有人try后都会变成loyalty,是有一定的条件的,例如之前使用过相同品牌的产品,等等
(4)A:证明plan很可行,很多消费者回冲着coupons去的,loser
B:plan不可行,消费者是冲着cents-off去的,try以后也不会变成loyalty,contender
C:无关选项,loser
D:不到1/3的coupons被回收了,与推出新品牌无关,loser
E:部分同<1>,但是并没说明只有cents-off就没效果,loser
所以,选B

5.
(1)45s weaken
(2)逻辑链
Background:
Many businesspeople who have not been to our state believe that we have an inadequate road system
Primes:
In each of the past six years, our state has spent more money per mile on road improvements than any other state.
Conclusion:
Those people are mistaken
(3)花的钱多不能证明就比别的state好,也可能是原来基础差所以现在要花更多的钱弥补
(4)A:增加的慢也是在增加,对argument没有影响,loser
B:无关比较,loser
C:无关信息,loser
D:无关信息,与修路花的钱和道路质量无关,loser
E:同预测,contender
所以,选E
作者: emmasy    时间: 2012-6-8 10:06
1.
事实:people have special knowledge are excluded from trials where the issue is revelant
结论:trial by jury is not a fair means of settling disputes involving such issues
a.support
b.weaken
c.irrevelant
d.irrevelant
e.irrevelant
作者: emmasy    时间: 2012-6-8 10:29
2.
事实:catfish swim erratically before earthquakes.
结论:some investigators have proposed monitoring catfish to predict dangerous earthquakes.
a.support
b.irrevelant
c.irrevelant
d.irrevelant
e.weaken conclusion

3.
事实:A city plans to attract new citizens with new housing and new facilities. One component of the city's plan is to require that developers seeking permission to build this new housing provide these additional facilities at no cost to the city.
a.weaken
b.irrevelant
c.support
d.irrevelant
e.support

4.
事实: manufacurers issue coupons to attract more consumers and hope that they will become loyal to the brand.
结论:In the initail marketing of X, manufacture should issue coupons
a.support
b.weaken事实
c.irrevelant
d.irrevelant
e.irrevelant

5.
事实:in each of the past six years, our state has spent more money per mile on road improvements than any other state
结论:we have an adequate road system
a.irrevelant
b.irrevelant
c.irrevelant
d.irrevelant
e.weaken事实
作者: bet    时间: 2012-6-18 00:18
1.     30”
People who have specialized knowledge about a scientific or technical issue are excluded from juries for trials where the issue is relevant.
C:trial by jury is not a fair means of settling disputes involving such issues.
预测:解决纠纷不需要专业知识,有了专业知识反而不能公正地解决问题
选B 其他选项无关

2  25”
P:catfish swim erratically before earthquakes
C:some investigators have proposed monitoring catfish to predict dangerous earthquakes.
预测:除了地震,其他原因也会造成catfish swim erratically
选E    AC加强 BD无关

3  40”
结论是什么??没读懂
读完选项才大概明白 貌似选A

4  26”
P:Manufacturers issue cents-off coupons to get consumers to switch their brand loyalty.
C:Hartman Industries should issue cents-off coupons to attract a large segment of potential consumers as loyal customers.
预测:很多其他因素会影响到brand loyalty
选B 直接否定结论

5  23”
P:in each of the past six years, our state has spent more money per mile on road improvements than any other state.
C:the road system of our state is adequate.
预测:之前路况太差,用钱多不意味着能真正提高路况
选E直接否定结论
作者: LuckyYolandaLi    时间: 2012-6-22 22:37
1
1)计时:71s
2)逻辑链:
Premise: People who have specialized knowledge about a scientific or technical issue are systematically excluded from juries for trials where the issue is relevant.
Conclusion: Trial by jury is not a fair means of settling disputes involving such issues.
3)推测:The exclusion from juries from trials is not because of unfairness.
4)选项分析:选C(正确答案B
B: Strengthen.(情何以堪,这是我第一个排除的答案,结果却是正确答案。。。)

(这题没太懂。。。)


2.
1)计时:71s
2)逻辑链:
Premise: Since catfish swim erratically before earthquakes, some investigators have proposed monitoring catfish to predict dangerous earthquakes.
Conclusion: To reduce the danger to life and property posed by major earthquakes, scientists have been investigating several techniques for giving advance warning of dangerous earthquakes.
3)推测:The prediction is not advanced enough that even though people get the prediction, they have no time to survive from the earthquake.
4)选项分析:选B(正确答案:E
E: Strengthen.(尴尬同上一题)


3.
1)计时:39s
2)逻辑链:
Premise: A city plans to attract new citizens with new housing and new facilities such as parks, recreation centers, and libraries.
Conclusion: One component of the city's plan is to require that developers seeking permission to build this new housing provide these additional facilities at no cost to the city.
3)推测:
4)选项分析:选A


4.
1)计时:46s
2)逻辑链:
Premise: Manufacturers issue cents-off coupons to get consumers to try their brand of product with the hope that the consumers who try their brand will switch their brand loyalty.
Conclusion: in the initial marketing of their new brand X, Hartman Industries should issue cents-off coupons, thereby attracting a large segment of potential consumers as loyal customers.
3)推测:
4)选项分析:选B
A: Strengthen.


5.
1)计时:33s
2)逻辑链:
Background information: Many businesspeople who have not been to our state believe that we have an inadequate road system.
Premise: It is obvious from the fact that in each of the past six years, our state has spent more money per mile on road improvements than any other state.
Conclusion: Those people are mistaken.
3)推测:4)选项分析:选E
作者: angelafeng    时间: 2012-6-25 22:49
今天有点晚了。。。
【精练】17s
1.



PremisePeople who have specialized knowledge about a scientific or technical issue are systematically excluded from juries for trials where the issue is relevant.



Conclusiontrial by jury is not a fair means of settling disputes involving such issues.
prephrase
:(weaken
answer
D----------B
(A) The more complicated the issue being litigated,
the less likely it is that a juror without
specialized knowledge of the field involved
will be able to comprehend the testimony
being given.--------------------this support the argument
(B) The more a juror knows about a particular
scientific or technical issue involved in a
trial, the more likely it is that the juror will be
prejudiced in favor of one of the litigating
parties before the trial begins.-----------------irrelevant--------correct,It agrees with the principle of
the premise, but not with the conclusion drawn from that premise.

(C) Appointing an impartial arbitrator is not a fair
means of settling disputes involving
scientific or technical issues, because
arbitrators tend to favor settlements in which
both parties compromise on the issues.------------the argument did not mention any information about arbitrator.
(D) Experts who give testimony on scientific or
technical issues tend to hedge their
conclusions by discussing the possibility of
error.--------------correct-----------This information does not attack the claim that jury trials are unfair because of the exclusion of jurors with.
knowledge of these issues.

(E) Expert witnesses in specialized fields often
command fees that are so high that many
people involved in litigation cannot afford
their services.---------------- the fees cannot weaken the conclusion of the arguement
没读懂题啊啊啊啊啊!
【逻辑链】
29. 27s
background information: To reduce the danger to life and property posed by major earthquakes, scientists have been investigating several techniques for giving advance warning of dangerous earthquakes.  



Premise: catfish swim erratically before earthquakes



Conclusion: some investigators have proposed monitoring catfish to predict dangerous earthquakes.
Prephrase: ( undermine)
没有地震时catfishswim erratically



Answer: E

30. 23s
premise
One component of the city's plan is to require that developers seeking permission to build this new housing provide these additional facilities at no cost to the city.  



ConclusionA city plans to attract new citizens with new housing and new facilities.
prephrase
:(flaw)开发商会抬高价格
answer
A







31.34s
background information
Manufacturers issue cents-off coupons to get consumers to try their brand of product with the hope that the consumers who try their brand will switch their brand loyalty.



Prephrase in the initial marketing of their new brand X, Hartman Industries should issue cents-off coupons



Conclusion attract a large segment of potential consumers as loyal customers.
prephrase
:(weaken)新品牌和成熟的品牌不能同日而语
answer: A-------------B



(A) Many consumers are unlikely to try new brands of products unless offered an inducement to do so.---------------this cannot weaken the argument
(B) The consumers whose purchases are strongly influenced by cents-off coupons tend not to become loyal customers of any particular brand.--------this directly dispute the argument about cents-off coupons
(C) Many grocery stores attract customers by doubling the face value of manufacturer's coupons.
(D) Typically less than one-third of the coupons issued by a manufacturer are redeemed by consumers.
(E) A marketing campaign that uses cents-off coupons is most effective when combined with a television advertising campaign.







32. 26s



Background informationMany businesspeople who have not been to our state believe that we have an inadequate road system.  



Premisein each of the past six years, our state has spent more money per mile on road improvements than any other state.



ConclusionThose people are mistaken
prephrase
:(weaken)花更多的钱改善道路说明这些道路原先有很多问题。



AnswerE






作者: 呵呵牙    时间: 2012-6-28 01:03
继续补作业~

1. 20s
Premise: People are excluded from juries for trials where scientific or technical issue is relevant.
Conclusion: Trial by jury is not a fair means of settling disputes involving such issues.
Prephrase: Trial by jury has its own way and involved other issues.
选A ——————support
B (prejudiced) ——support ————————correct
C (appointing an impartial arbitrators) ——irrelevant
D (hedge conclusion)——irrelevant
E (fees)——irrelevant

2. 22s
Background Information: Scientists have been investigating several techniques for giving warning of earthquakes to reduce the danger.
Premise: Catfish swim erratically before earthquakes.
Conclusion: Some investigators have proposed monitoring catfish to predict dangerous earthquakes.
Prephrase: If catfish are in danger, maybe they also swim erratically.
选E

3. 20s
Premise: The plan is require that developers seeking permission to build new housing provide these addition facilities at no cost to the city.
Conclusion: A city plan to attract new citizens with new housing and facilities.
Prephrase: This plan will cost much money; people cannot pay attention to the new housing and facilities.
选A

4. 36s
Premise: Manufacturers use coupons to get more consumers buy their products and become loyal customers.
Conclusion: Hartman Industries issue coupons to sell their new brand X and attract potential consumers as loyal customers.
Prephrase: People buy the new product just because they have coupons; if other brand also issue coupons, the consumers will buy other products.
选B

5. 30s
Premise: Many businesspeople believe that we have an inadequate road system.
Conclusion: In each of the past 6 years, our state has spent more money per mile on the road improvement than others.
Prephrase: The road system in this state is more faster advancing than others.
选E
作者: Donts    时间: 2012-7-15 11:24
1.计时18''
premise:a expert on some issues is systematically excluded from a jury of trails.conclusion:the trail of jury is not a fair way to settle disputes involving such issues.
推测:这些issue不仅需要专业人士。
(A) The more complicated the issue being litigated,
the less likely it is that a juror without
specialized knowledge of the field involved
will be able to comprehend the testimony
being given.这好像是加强。
(B) The more a juror knows about a particular
scientific or technical issue involved in a
trial, the more likely it is that the juror will be
prejudiced in favor of one of the litigating
parties before the trial begins.正确。
(C) Appointing an impartial arbitrator is not a fair
means of settling disputes involving
scientific or technical issues, because
arbitrators tend to favor settlements in which
both parties compromise on the issues.加强
(D) Experts who give testimony on scientific or
technical issues tend to hedge their
conclusions by discussing the possibility of
error.不影响fairness
(E) Expert witnesses in specialized fields often
command fees that are so high that many
people involved in litigation cannot afford
their services.fees无关
总结:
Answer choice (C): This answer simply notes that arbitrators are not a fair means of settling scientific ortechnical issue debates. This has no impact on the fairness of jury trials involving these same issues.
作者: showysimple    时间: 2012-7-29 17:33
1.

[Premise] – ppl who have specialized knowledge are excluded from juries
[Conclusion] – trial by jury is not a fair means of settling disputes involving such issues

求weaken
[Prephrase] – the exclusion is a fair way to settle such trials

(A) support prime only
(B) right answer
(C) ‘Arbitrator’ is irrelevant, 讨论的是jury trial
(D) ‘Experts who give testimony on scientific or technical issues’ is irrelevant, 讨论的是jury trial
(E) ‘expert witnesses in specialized fields’ is irrelevant, 讨论的是jury trial

2.

[Premise] – catfish swim erratically before earthquakes
[Conclusion] – monitoring catfish is proposed to predict dangerous earthquakes

weaken
[Prephrase] – 测鱼不足以准确地预测地震,因为catfish在其它情况下也swim erratically?

(A) supportive
(B) ‘mechanical methods of detecting earthquakes’ is irrelevant, 讨论的是 ‘monitoring catfish’
(C) 为什么震颤会导致鱼类不规则地游动与题目无关
(D) 讨论的是如何预测大型地震,而非地震过后的影响
(E) attack premise, right answer

5.

[Premise] in each year of the past 6 years, this state spent more money per mile on road impromt. than any other state
[Conclusion] bizppl think there’s an inadequate road system in this state, it’s wrong

weaken
[Prephrase] – 投资在铁路建设上=铁路系统不完善?

(A) – irrelevant
(B) – irrelevant
(C) – irrelevant
(D) – irrelevant
(E) – attack premise, right answer
作者: TICKCOCK    时间: 2012-8-7 20:04
1.    remise: jury exclude people specialized in scientific and technical issue.
Conclude: trial by jury is not a fair means of settling the disputes of such issue.
Answer: B

2.    To reduce the danger earthquake poses to life and property, scientists have been investigating several techs to predict earthquake.
Premise: catfish swim erratically before earthquakes.
Conclusion: some investigators have proposed monitoring catfish to predict the earthquake.
Answer: E

3.    City plan: to attract new citizens with new housing and new facilities.
One component contends the city would cost nothing because of these facilities.
Answer: D (A)

4.    remise: manufacturers issue coupon to get customers to try their brand of product hoping customers switch their brand loyalty.
Conclusion: Hartman Industries should issue coupons to attract customers for Brand X.
Answer: B

5.    State spokesperson: it is mistaken for people who have never been to the state to think the instate road system is inadequate.
Reason: the state has spent more money per mile on road improvements than any other states.
Prephase: road improvement is not equal to adequate road systems.  Answer: E
作者: chengzaaaa    时间: 2012-8-26 00:29
1.
1)21”weaken
2)BI:
P: people, know about scientific or tech issues, excluded from jury
C: such issues, not fair, by jury
3)knowing the relevant things would affect the judgement. Other people judging is more fair.
4)B
5)A. wrong comparison
B. Correct. 先入为主 and they can’t help it.
C. not weakening the reasoning in the argument.
D. irrelevant
E. irrelevant

29.
1)17”weaken
2)BI: want earthquake prediction, working on that
P: catfish act weird before earthquakes
C: use catfish to detect
3)teeny movement, not earthquake, would make catfish move a lot. A lot false alarm
E

30.
1)20”weaken
2)BI:
P: additional stuff with no cost to the city
C: new house with new facilities to attract new citizens
3)not possible to build new facs without cost to city.
A

31.
1)24”weaken
2)BI: Manus issue coupons to attract new people, hoping they get loyal
P:
C: X send coupons to attract, loyal
3)coupon not making them loyal, they try everything’s coupons.
B

32.
1)17”weaken
2)BI: people say inadequate road system
P: past 6 years, spent more on road improvements than other states
C: they are wrong
3)beginning of the building of the road system
E


谢谢lz!!!
作者: 我心匪席    时间: 2012-9-1 18:55
Time: 24’
Premises: People who has special knowledge will be excluded from trial, where issue is relevant
Conclusion: The means to settle down such issues are not fair.
Prephrase: Trial is irrelevant
A-----support
B-----right weaken
C-----irrelevant
D-----irrelevant
E-----irrelevant
============================================
Time: 21’
Background Informant: Earthquake cause damage
Premises: Catfish swim erratically before earthquake
Conclusion: Monitor catfish can predict earthquake
Prephrase: Other condition can cause catfish swim erratically
A-----irrelevant
B-----irrelevant
C-----irrelevant
D-----irrelevant
E-----right weaken
===============================================
Time: 23’
Premises: Developer can build new facilities at no cost to city
Conclusion: City plans to attract new citizens with new facilities
Prephrase: Cost is essential
A-----right weaken
B-----irrelevant
C-----irrelevant
D-----irrelevant
E-----irrelevant
==============================================
Time: 41’
Background Information: Cents-off coupons attract consumers
Premises: X use cents-off coupons
Conclusion: Attract potential customers
Prephrase: Loyalty can not easily be changed by cents-off coupons.
A-----support
B----- right weaken
C-----irrelevant
D-----irrelevant
E-----irrelevant
===========================================
Time: 25’
Background Information: Many business people believe state has an inadequate road system
Premises: Spend more money per mile on road improvements than other state.
Conclusion: Mistaken
Prephrase: Cost does not relate to quality.
A-----irrelevant
B----- irrelevant
C-----irrelevant
D-----irrelevant
E-----right weaken
作者: srafcatt    时间: 2012-10-7 12:51
精炼15s weaken

because:experts are not allowed in juries

thus:trial is not a fair means in relevant issues



trails represent the majority,so it is fair.

choice:B

A if the trails cannot comprehend the complicated issue then they are likely to make wrong decisions.So this choice strenghten the argument

B this is correct,because of this reason,the trail need to be consisted of normal people.

C this choice strenghten the information listed in the question stem

D this is an irrelevant argument.

E irrelevant information,even if the experts charge a significantly high fees,it does not mean that the trails are not fair.



逻辑链

1.33s weaken

because:catfish swim erratically before earthquakes,some earthquake predictions by them have succeeded.

thus:the danger to life and property can be reduced.



some cases succeed cannot ensure all the time it will work.

choice:C

A strenghten the argument

B irrelevant information

C weaken the argument,since all the aquatic animals will behave erratically,there is no need to observe and only observe the catfish

D irrelevant information

E strenghten the argument





2.39s flaw

premise:the facilities are free

then:the city plan to attract new citizens will succeed.



flaw:even if the facilities are free of charge,the payment to construct them and later fees to mend them will cost the city a lot.

choice:A

A correct,this fact means there virtually is no use of the free facilities.

B strenghten the plan

C irrelevant,the question stem is about the plan,nothing to do with the competitors.

D strenghten the argument by saying that the developers tend to offer free facilities.

E repeat the information in the question stem



3.35s weaken

BG:it is possible to succeed switching the loyalty by issue the cents-off coupon

if:a new brand in initial market wants more customers

then:the manufacturer should issue that coupon.



How about customers who don't care about money and cherish more about quality?

choice:B

A strenghten

B weaken ,correct answer

C although doubling the face value of the coupons,it simply means that the cost will be enlarged but the loyalty has been switched

D but someone's loyalty is still being switched,that hits the target.

E irrelevant information



4.23s weaken

BG:foreighner considers that this country lack of road system

because: spend more money than other states during the past six years

then: foreigners are wrong



more money does not equal quality



choice

A irrelevant,as long as the road system now is great,no one cares about the time that procedure takes.

B irrelevant

C irrelevant

D correct,weaken

E the current situation is the most important.



作者: srafcatt    时间: 2012-10-7 13:05
纠错:逻辑链第一题:E项的意思是即便是十分微小的晃动也会使鲶鱼不安,B项不对是因为别的动物可以预测地震不代表鲶鱼不行。典型的无关信息。

逻辑链最后一题:预测的时候就在想是不是质量不行,即便有公路系统也被认为是不好的。我忽略的是逻辑题要求单词的意思掌握要精确,题目中说的是不充足的道路系统,是量上的不充足。和质量没关系。

下次再做题,一定要反复审题,看好题目的意思再下结论
作者: hanhan1991    时间: 2012-11-20 23:08
1.weaken
1)time:31s/2''16'
2)logical lines:
Premiseeople who have specialized knowledge about a scientific or technical issues are systematically excluded from juries for trails where the issue is relevant.
Conclusion:Trial by jury is not a fair means of settling dispute involing such issues.
3)Predication:
Even if the people of professional knowledge are excluded from the jury, the jury can fairly deal with those issues.
4)Answer:B
A.this strengten the argument.
B.since the professional knownledge affects the attitute of the jury, excluding these people can make the trail more fair.
C.the argument do not concerns about appointing an impartial arbitrator.
D.it can be a contender. however, discussing the possibility of error does not affect the fairness of the trail, so it is irrelevant.
E.fee is irrelevant.

2.weaken
1)time:25s/58'
2)logical lines:
Background:catfish swim erratically before earthquakes
Objective:To predict the earthquakes in advance
Plan: Monitoring the action of catfish
3)Predication:
Monitoring the action of catfish can not predicat the earthquake in advance, since the catfish may swim erratically even when there is no eathquake.
4)Answer:C
A.this strengthen the argument.
B.this irrelevant.
C.it says that the hydrogen sulfide gas but not the tremors cause the catfish swim erratically, which weakens the argument.
****the hydrgen sulfide gas in the water is caused by the temor, so it is the temor caused the catfish swim erratically.
D.irrelevant.
E.this strengthen the argument
****because we can not ditinguish sightly temor from earthquake by the method so the argument can be weakened.

3.flaws
1)time:31s/1''34'
2)logical lines:
Objective:Attract new citizens with new housing and new facilities
Plans:Requrie that developer seeking permission to build this new housing provide these additional facilities at no cost to the city.
3)Prediction:
The proposal might drecrease developers' incentives to build in the city and thus result in fewer houses and facilities.
4)Answer:A
A.If the citizens can not afford to buy a house in the city, they would leave to other cities so the plan is flawed.
B.this is irrelevant.
C.other towns is irrelevant.
D.this is irrelevant or even strengthen the argument.
E.this strengthen the argument.

4.weaken
1)time:30s/1''08'
2)logical lines:
Background:Manufacturers issue cent-off coupons to get consumers to try their product with the hope that the consumers who try their brand will switch their brand loyalty.
Objective:To attract a large segment of potential consumers as loyal customers
Plan:Issue cent-off coupons of thei new brand X.
3)Prediction:
People see products with the cent-off coupons as low quanlity and thus will not buy them in the future.
4)Answer:B
A.this strengthen the argument.
B.even if the store offers the cent-off coupons to its customers, they would not become loyal to the store,which weakens the argument.
C.this is relevant or even strengthen the arguement.
D.it is irrelevant.
E.it is irrelevant.

5.weaken
1)time:21s/57'
2)logical lines:
Premise: The city spends much more money on road improvement than any other cities.
Conclusion: The belief that the road system is poor in this city is flawed.
3)Predication:
Spend more money on road improvement does not equal to the city has adequate road system, it is possible that only the city with poor road system need to spend much more money on the road improvement.
4)Answer:E
A.the icreasing rate of spend on road improvement is irrelevant.
B.it is irrelevant.
C.it is irrelevant.
D.it is irrelevant.
E.it is the same as the prediction.
作者: CD用户825193    时间: 2013-1-5 18:08
标题: Daily CR-7_2013-01-05
1. People who have specialized knowledge about a
scientific or technical issue are systematically
excluded from juries for trials where the issue is
relevant. Thus, trial by jury is not a fair means of
settling disputes involving such issues.
Which one of the following, if true, most seriously
weakens the argument?
20”+18“+17" (Weak) 【可以先不计时来练习嘛?每次计时都心慌意乱的啊,老是看不清题目。。。o(╯□╰)o】
P:  People have relative knowledge r excluded to such injuries. juries!!!
     eople have specific knowledge of the issues are excluded from juries.
C: Trials include those people are not fair means.
     Trial excluded those people (for such issues) is not fair means.
Pre: There r always a group of people with specific knowledge of some topics
     Specific knowledge of the issues generally make people's judgement unjust.

Answers:
A) The more complicated the issue being litigated,
the less likely it is that a juror without
specialized knowledge of the field involved
will be able to comprehend the testimony
being given.
Here we talk about all issues, nothing about the level of being complicated. [Wrong analysis... this should support the argument]

(B) The more a juror knows about a particular
scientific or technical issue involved in a
trial, the more likely it is that the juror will be
prejudiced in favor of one of the litigating
parties before the trial begins.
CORRECT: Specific knowledge influence a juror's judgement even before the trial.

(C) Appointing an impartial arbitrator is not a fair
means of settling disputes involving
scientific or technical issues, because
arbitrators tend to favor settlements in which
both parties compromise on the issues.
"appointing an impartial arbitrator" is irrelevant to the topic.

(D) Experts who give testimony on scientific or
technical issues tend to hedge their
conclusions by discussing the possibility of
error.
Kinda support the argument, not weaken. [Not relevant to the topic]

(E) Expert witnesses in specialized fields often
command fees that are so high that many
people involved in litigation cannot afford
their services.
"command fees" is irrelevant to the topic.
2. To reduce the danger to life and property posed by major earthquakes, scientists have been investigating several techniques for giving advance warning of dangerous earthquakes.  Since catfish swim erratically before earthquakes, some investigators have proposed monitoring catfish to predict dangerous earthquakes.
Which of the following, if true, most seriously undermines the usefulness of the proposal?
20" (Weak)
P: A fish swim erratically before earthquake.
C: Scientists try to use the activity of the fish to warn ppl of the coming earthquake in advance
Pre: The fish only starts to act erratically short before the earthquake, so not enough time for ppl to get rid of the danger.
The fish acts erratically for many reasons, so it's possible that the earthquake is not the reason.
It's hard to observe the fish's activity being erratically?

Answers:
(A) In Japan, which is subject to frequent earthquakes, the behavior of catfish has long been associated with earthquakes.
   Support.
(B) Mechanical methods for detecting earthquakes have not proved effective.

   No fish mentioned, irrelevant.
(C) Tremors lead to the release of hydrogen sulfide gas into water, thereby causing various fish and shellfish to behave erratically.

   It's the reason of fish behave erratically, doesn't harm the argument. Irrelevant.
(D) Careful construction can reduce the dangers posed by earthquakes.

   No fish mentioned. irrelevant.
(E) Even very slight, fleeting tremors cause catfish to swim erratically.

   CORRECT. Just as one pre i mentioned, hahaha...

3.A city plans to attract new citizens with new housing and new facilities such as parks, recreation centers, and libraries.  One component of the city's plan is to require that developers seeking permission to build this new housing provide these additional facilities at no cost to the city.
Which of the following, if true, would point to a possible flaw in the city's plan?

25" (Weak: possible flaw)
P: Government is trying to attract new citizens with more/better public facilities, such as park, libraries...
P: Government requires housing developers to provide more public facilities without cost of the city.
C:
Pre: Developer will transfer the cost to housing buyers -> house price gets high -> less ppl want to move over

Answers:
A) Developers would pass along their costs to the buyer, thereby raising the cost of housing units beyond the ability of likely purchasers to afford them.
   ?CORRECT,
but not sure, as no new citizens mentioned. (after reading all other choices, sure of this choice)
(B) Light, nonpolluting industries have located in the area, offering more jobs and better-paying jobs than do the more-established industries in the area.
   Irrelevant with the housing developer plan.
(C) Other towns and cities nearby have yet to embark on any comparable plans to attract new citizens.
   Other towns r irrelevant.
(D) Most developers see the extra expense of providing municipal facilities as simply one of the many costs of doing business.
   Not a flaw.
(E) Studies show that purchasers of new houses, especially first-time buyers, rank recreational resources as an important factor in deciding to buy a particular house.
   Not a flaw.

4. Manufacturers issue cents-off coupons to get consumers to try their brand of product with the hope that the consumers who try their brand will switch their brand loyalty.  So in the initial marketing of their new brand X, Hartman Industries should issue cents-off coupons, thereby attracting a large segment of potential consumers as loyal customers.
Which of the following, if true, casts the most serious doubt on the likelihood that the marketing strategy recommended above will have the result that is claimed?

27" (Weak)
P: Free sample can attract potential consumers to change the brand choice and be loyal to the other brand. [I didn't notice if this theory works for all brands or a new brand.]
[F... it was about the cents-off coupons, why I remembered it as free samples? oh, oh, I just got an email inviting me to use their free samples!!!]
C: X Industries should send free samples to get other brand using consumers being loyal to its brand
Pre: ?? The theory doesn't work for new brand.
   Too many free sample will make consumers think the product/brand cheap.
   [Cents-off coupons will harm the profit badly?]

Answers:
(A) Many consumers are unlikely to try new brands of products unless offered an inducement to do so.
   Support
(B) The consumers whose purchases are strongly influenced by cents-off coupons tend not to become loyal customers of any particular brand.

   CORRECT.
(C) Many grocery stores attract customers by doubling the face value of manufacturer's coupons.

   Irrelevant
(D) Typically less than one-third of the coupons issued by a manufacturer are redeemed by consumers.

   Irrelevant
(E) A marketing campaign that uses cents-off coupons is most effective when combined with a television advertising campaign.


   Support


5. State spokesperson:  Many businesspeople who have not been to our state believe that we have an inadequate road system.  Those people are mistaken, as is obvious from the fact that in each of the past six years, our state has spent more money per mile on road improvements than any other state.
Which of the following, if true, most seriously undermines the reasoning in the spokesperson's argument?
26" (Weak)
P: Ppl have not been to the state think the road system in the state is inadequate.
P: In the past six years, the state spent more money on road system than any other states.
C: The road system in the state is not inadequate as some ppl said.
Pre: More money ≠ adequate: it might be much worse six years ago, so now even more effort on the system, it's still not good enough.                           spending more money doesn't mean road system improvement

Answers:
(A) In the spokesperson's state, spending on road improvements has been increasing more slowly over the past six years than it has in several other states.
   CORRECT.
[** Increasing slowly doesn't mean inadequate...]

(B) Adequacy of a state's road system is generally less important to a businessperson considering doing business there than is the availability of qualified employees.
   Irrelevant.
(C) Over the past six years, numerous businesses have left the spokesperson's state, but about as many businesses have moved into the state.

   Irrelevant.
(D) In general, the number of miles of road in a state's road system depends on both the area and the population of the state.

   no money spending on the road system mentioned, so irrelevant.
(E) Only states with seriously inadequate road systems need to spend large amounts of money on road improvements.

   Irrelevant to the road system's quality.
[CORRECT, feel like typical GMAT logic... should read and understand the choice more carefully...]


作者: zxppx    时间: 2013-1-11 14:42
1.

People who have specialized knowledge about a
scientific or technical issue are systematically
excluded from juries for trials where the issue is
relevant. Thus, trial by jury is not a fair means of
settling disputes involving such issues.
Which one of the following, if true, most seriously
weakens the argument?

The argument involves in the line of reasoning that since people with specialized knowledge about a scientific or technical issues are excluded from juries, the trial by jury is not a fair means. Thus, in order to weaken the argument, we need to point out the reason why these people should be excluded from juries for trials, instead of keeping them in the juries. The real challenge of the problem is that the right answer seems to cooperate with the premise, but the truth is that the answer calls question into the conclusion.
Weaken Question Problem Set NO.9

A) The more complicated the issue being litigated,
the less likely it is that a juror without
specialized knowledge of the field involved
will be able to comprehend the testimony
being given

If a juror cannot comprehend the testimony being given, the trial by jury is not a fair means. So a juror needs to comprehend complicated issue by owning specialized knowledge about a scientific or technical issue. On the other hand, we can say that having specialized knowledge about a scientific or technical issue is helpful for jury to make the trial fair. BA

I mistook the meaning of the argument, and the choice A is the opposite answer that strengthens the argument.
(B) The more a juror knows about a particular
scientific or technical issue involved in a
trial, the more likely it is that the juror will be
prejudiced in favor of one of the litigating
parties before the trial begins.

Since the prejudice in the trail, the answer strengthens the argument.

It is the right answer, since it points out the reason why some experts should be excluded from the juries.
(C) Appointing an impartial arbitrator is not a fair
means of settling disputes involving
scientific or technical issues, because
arbitrators tend to favor settlements in which
both parties compromise on the issues.

Shell Game: the argument talks about having a special knowledge about a particular issue, rather than appointing an impartial arbitrator. Even though the conclusion refers to fairness, it does not matter with the fact that arbitrators tend to favor settlements in which both parties compromise on the issue.
(D) Experts who give testimony on scientific or
technical issues tend to hedge their
conclusions by discussing the possibility of
error.

The argument has no relationship with the way experts used to hedge their conclusions.
(E) Expert witnesses in specialized fields often
command fees that are so high that many
people involved in litigation cannot afford
their services.

It cannot solve the fairness referred in the argument.



29.
To reduce the danger to life and property posed by major earthquakes, scientists have been investigating several techniques for giving advance warning of dangerous earthquakes. Since catfish swim erratically before earthquakes, some investigators have proposed monitoring catfish to predict dangerous earthquakes.
Which of the following, if true, most seriously undermines the usefulness of the proposal?

(A) In Japan, which is subject to frequent earthquakes, the behavior of catfish has long been associated with earthquakes.

It strengthens the argument.
(B) Mechanical methods for detecting earthquakes have not proved effective.

It does not weaken the argument.
(C) Tremors lead to the release of hydrogen sulfide gas into water, thereby causing various fish and shellfish to behave erratically.

The answer talks about the reason why some fish and shellfish would behave erratically, actually strengthening the argument.
(D) Careful construction can reduce the dangers posed by earthquakes.

It does not refer to the catfish.
(E) Even very slight, fleeting tremors cause catfish to swim erratically.

If the fact is true, the way of catfish to measure earthquake is not valid, since the way is not precise. BA



30.

A city plans to attract new citizens with new housing and new facilities such as parks, recreation centers, and libraries. One component of the city's plan is to require that developers seeking permission to build this new housing provide these additional facilities at no cost to the city.
Which of the following, if true, would point to a possible flaw in the city's plan?

(A) Developers would pass along their costs to the buyer, thereby raising the cost of housing units beyond the ability of likely purchasers to afford them.

If purchaser are less likely to pay for the fees about these new facilities, the plan are more likely to fail. BA
(B) Light, nonpolluting industries have located in the area, offering more jobs and better-paying jobs than do the more-established industries in the area.

It strengthens the argument.
(C) Other towns and cities nearby have yet to embark on any comparable plans to attract new citizens.

The same to B.
(D) Most developers see the extra expense of providing municipal facilities as simply one of the many costs of doing business.

The same to B.
(E) Studies show that purchasers of new houses, especially first-time buyers, rank recreational resources as an important factor in deciding to buy a particular house.

The same to B.



31.
Manufacturers issue cents-off coupons to get consumers to try their brand of product with the hope that the consumers who try their brand will switch their brand loyalty. So in the initial marketing of their new brand X, Hartman Industries should issue cents-off coupons, thereby attracting a large segment of potential consumers as loyal customers.
Which of the following, if true, casts the most serious doubt on the likelihood that the marketing strategy recommended above will have the result that is claimed?

(A) Many consumers are unlikely to try new brands of products unless offered an inducement to do so.

Since the Hartman Industries have offered coupons, an inducement for consumers to try new brands of products, the answer does not weaken the argument.
(B) The consumers whose purchases are strongly influenced by cents-off coupons tend not to become loyal customers of any particular brand.

If the answer is true, the argument is heavily weakened, since the Hartman Industries cannot attract enough potential consumers as loyal customers. BA
(C) Many grocery stores attract customers by doubling the face value of manufacturer's coupons.

Shell Game: there is no relationship with grocery stores.
(D) Typically less than one-third of the coupons issued by a manufacturer are redeemed by consumers.

Even though the proportion is not great, it does not mean that brand X could not attract loyal consumers.
(E) A marketing campaign that uses cents-off coupons is most effective when combined with a television advertising campaign.

Even though the coupons alone may be not the most effective, it does not make sure that the plan would fail.

32.
State spokesperson: Many businesspeople who have not been to our state believe that we have an inadequate road system. Those people are mistaken, as is obvious from the fact that in each of the past six years, our state has spent more money per mile on road improvements than any other state.
Which of the following, if true, most seriously undermines the reasoning in the spokesperson's argument?

(A) In the spokesperson's state, spending on road improvements has been increasing more slowly over the past six years than it has in several other states.

Even though the spending is slower over the past six years than it was in several other states, the fact does not reveal that this state has not an adequate road system.
(B) Adequacy of a state's road system is generally less important to a businessperson considering doing business there than is the availability of qualified employees.

There is no relationship with the comparison between the adequacy of a state’s road system and the availability of qualified employees.
(C) Over the past six years, numerous businesses have left the spokesperson's state, but about as many businesses have moved into the state.

It does not undermine the argument.
(D) In general, the number of miles of road in a state's road system depends on both the area and the population of the state.

It does not matter with what decides the number of miles of road in a state’s road system.
(E) Only states with seriously inadequate road systems need to spend large amounts of money on road improvements.

This answer rightly points out the reason why the state has spent more money per mile on road improvements than any other state, thus weakening the argument. BA

作者: pennyz    时间: 2013-2-10 23:12
又要拖一天了,叫我拖拖吧!
作者: pennyz    时间: 2013-2-11 22:37
1:59
d
premise:the experts were exclude
conclusion:the disscuss is not fair
rephrase:the expert's idea may be useless
A) The more complicated the issue being litigated,the less likely it is that a juror without specialized knowledge of the field involved will be able to comprehend the testimony being given.-----strengthen,since the decision without expert tends to be wrong
(B) The more a juror knows about a particular scientific or technical issue involved in a
trial, the more likely it is that the juror will be prejudiced in favor of one of the litigating parties before the trial begins.------?shortcomings?
(C) Appointing an impartial arbitrator is not a fair means of settling disputes involving
scientific or technical issues, because arbitrators tend to favor settlements in which
both parties compromise on the issues.-------irrelevant
(D) Experts who give testimony on scientific or technical issues tend to hedge their
conclusions by discussing the possibility of error.-----experts' shortcomings
(E) Expert witnesses in specialized fields often command fees that are so high that many
people involved in litigation cannot afford their services.------irrelevant
作者: pennyz    时间: 2013-2-11 23:05
1:12s weaken
E
premise: c swim erratically before earthquake
conclusionbserve c to forecast earthquake
rephrase:it's hard to detect the action of c
        c may swim erratically because of the reason other than earthquake

2:00s
a
aim:to attract new citizens by new housing and new facilities
methods:ask constructures to build facilities for free
rephrase:free facilities may cause rise in house price
1:30s
b
aim:to lure more potential customers and switch them into loyal consumers
methods: by giving off coupons
rephrase:the consumer may not use these coupons
1:43sweaken
business :fewer roads in the state
state:we spend more money on road improvement than others
rephrase:improvement only on the existence road,no add of the new roads
e
作者: 一粒黄豆。    时间: 2013-3-12 23:33
3/12 【1-8】
30’
premise: 有专业知识的人被 excluded from juries
conclusion:trial by jury is not a good means
prephrase:专业知识的人有xxx不好
A 把范围缩小到complicated issue,跟原本重点不一样
B 说了有专业知识的缺点 correct
C 重点说的是arbitrator,跟题干重点不一样
D 说的是give testimony的人,irrelevant
E 讲的是fee,重点错误

-----〉B

28’
premise:catfish swim erratically before earthquakes
conclusion: monitoring catfish to predict dangerous earthquakes
prephrasether factors can also cause the same phenominon
A irrelevant
B irrelevant
C various fish and shellfish貌似无关,但是erratically还是有关的,留着
D irrelevant
E erratically有关,但是这个到底是在support还是weaken呢?

对比CE还是决定选C,因为various fish也包括catfish,导致catfish的行为不能准确预测地震
E好像是说catfish精确度很好,像是support

初选-----〉C

好吧,这道题果然还是错了。。。不过我还是不太能确定E到底是正面还是负面,slight到底是强调他灵敏呢,还是过敏呢?
C可能还是irrelevant面比较大~

正确答案--------〉E

36’
info:city want to attract new citizen with new housing and new facilities
part of the plan require housing developers provide additional facilities

housing+facilities--->1attract new citizen ;2have facilities no cost to city

prephrase:flaw:
1poor quality,can't attract citizens
2 cause other cost to city
A 最终导致can't attract citizens
B job,irrelevant
C other town跟这个计划无关
D support
E support
----->A


33'
premise:manufacturers issue coupin to get loyal customers
conclusion:in the initial marketing of new brand X, HI should issue coupons to attract...
prephrase:is there any difference between brand of product and new brand X? or is there any...between manufacturers and HI?
in other words:some factors may cause HI to fail
A 重点好像有点偏,讲的是怎么改善,而不是coupon本身的问题
B 看起来像正确答案,顾客不loyal
C 其他grocery irrelevant
D 这个是结果,题干说的是建议,而且少于1/3也不能定性为不好
E 什么时候最effective,out of scope
-----〉B

23’
premise:cost more than other states on road in past six years
conclusion:people are mistaken in believing that we have an inadequate road system
prephrase:cost more doesn't mean better, road system is determined by both improvement and initial conditions
A slowly ,irrelevant
B business ,irrelevant
C business,irrelevant
D number of miles of road, irrelevant
E correct
----->E
作者: 一粒黄豆。    时间: 2013-3-12 23:43
逻辑链第一题:
premise:catfish swim erratically before earthquakes
conclusion: monitoring catfish to predict dangerous earthquakes
prephrasether factors can also cause the same phenominon
重新看了一下C选项!的确是错的
C重心在前半句:说的是地震引起海洋动物erratically的原因是blablabla,跟catfish能不能用来预测地震无关!!
所以说阅读很重要啊,一个词都不能漏啊!
作者: 纽约我爱你    时间: 2013-3-29 16:38
1~41s
Premise: people who know much about the issue are excluded out of it
Conclusion: it’s unfair for the issue.
Weaken: people who know more, bias more.
Answer: B
2~34s
Premise: the catfish swim awkward before earthquake
Conclusion: scientists want to monitor catfish to predict earthquake
Weaken: 1.the fish won’t do so under monitoration
2.The fish don’t do so only before an earthquake
Answer: E
3~45s
Premise: the city is going to build some facilities to attract new citizens
Conclusion: the plan wants to build them without cost to the city
Flaw: 想、想不出来……orz这两者我怎么觉得没关系的……
AnswerA 1’21’’
4~41s
Premise: 1.the cent-off strategy is used to make the new-buyers loyal
2.the manager of a new production uses this strategy
Conclusion: he hopes to attract new consumers.
Weaken: 1.once he stop the strategy, the consumers will leave
2.other companies may use the same strategy
Answer: D 2’07’’
正解:B
这个题我看了有一阵子,问题主要还出在理解上。阅读能力是硬伤……
5~19s
Premise: the state spent more money on road than any other states did
Conclusion: the idea that it has inadequate road system is wrong
Flaw: 1. the money are spent on the repairation of the road
2.the labor at that state is expensive, the money are just for the pay
Answer: E 44s

作者: okplokpl0714    时间: 2013-4-1 22:56
精练:
People excluded from juries for trials where issue is relevant-->trial by jury is not a fair means
题目看不懂,先蒙一个答案吧:A
重新分析:
Pre:People who have specialized knowledge about a
scientific or technical issue are systematically
excluded from juries for trials where the issue is
relevant.
Con:trial by jury is not a fair means of
settling disputes involving such issues.
Which one of the following, if true, most seriously
weakens the argument
前提:拥有科学或技术的专业知识的人被系统性地排除在与其知识无关的陪审团外
结论:陪审团审讯不是一个公平的举动
削弱答案:B
在陪审团里的陪审员涉及到科学或技术专业知识越多,陪审员在陪审开始前受到偏见的可能性越大




逻辑链:
29.scientist invest techniques for advance warning of dangerous to reduce danger
catfish swin erratically before earthquakes-->propose catfish to predict earthquakes
削弱猜测:catfish有自身缺陷,不能完全预测地震
Answer:C

30.city plans attract new citizen with new facilities.
One of plan is to seek permission no cost
削弱猜测:这个计划肯定有所代价
Answer:E
答案是A,之前没有看懂。。。猜测方向是对的。。

31.coupons-->hope consumer loyalty
So initial of new brand X ,HI should coupon
削弱猜测:coupon策略在某些情况下失效
Answer:B

32.business people not to our state beileve we have inadequate road system.
each of the past six years,our state spend more on road improvement-->those people are mistaken
削弱猜测:花得多不说明路好
Answer:E



作者: Feelalive    时间: 2013-7-8 18:02
路过~~~~~~~~~~~
作者: Mint静默    时间: 2013-7-13 10:53
1.People who have specialized knowledge about a
scientific or technical issue are systematically
excluded from juries for trials where the issue is
relevant. Thus, trial by jury is not a fair means of
settling disputes involving such issues.
Which one of the following, if true, most seriously
weakens the argument?
计时:25’
Premise: Expertised people are excluded from the juries for trials.
Conclusion: Trials by jury is not a fair means of setting disputes involving such issues.
推测(weaken)
People who have specialized knowledge 容易产生bias. 对 the judgment有害。
(A) The more complicated the issue being litigated,
the less likely it is that a juror without
specialized knowledge of the field involved
will be able to comprehend the testimony
being given.
Support
(B) The more a juror knows about a particular
scientific or technical issue involved in a
trial, the more likely it is that the juror will be
prejudiced in favor of one of the litigating
parties before the trial begins.
Correct
(C) Appointing an impartial arbitrator is not a fair
means of settling disputes involving
scientific or technical issues, because
arbitrators tend to favor settlements in which
both parties compromise on the issues.
impartial arbitrator不知是指谁,若是指expertised people就可以
(D) Experts who give testimony on scientific or
technical issues tend to hedge their
conclusions by discussing the possibility of
error.
审慎的对待自己的结论应该更加fair 才对
(E) Expert witnesses in specialized fields often
command fees that are so high that many
people involved in litigation cannot afford
their services.
Fees不在讨论范围内
T2 22’ 选E
T3 20’ 选A
T4 23’ 选B
T5 17’ 选E

作者: Elisha728    时间: 2013-8-29 06:44
5'46''BEABE
作者: lyrsilvia    时间: 2013-9-15 22:36
B EABE
1. 25‘
P: .People who have specialized knowledge about a scientific or technical issue are systematically excluded from juries for trials where the issue is relevant.
C: trial by jury is not a fair means of settling disputes involving such issues.
Weaken: exclude makes more fair
答案:B
(A) The more complicated the issue being litigated, the less likely it is that a juror without specialized knowledge of the field involved will be able to comprehend the testimony being given.----------STRENGTHEN
(B) The more a juror knows about a particular scientific or technical issue involved in a trial, the more likely it is that the juror will be prejudiced in favor of one of the litigating parties before the trial begins.----------point out the disadvantage to include expert. WEAKEN,CORRECT
(C) Appointing an impartial arbitrator is not a fair means of settling disputes involving scientific or technical issues, because arbitrators tend to favor settlements in which both parties compromise on the issues.---------------impartial arbitrator is not appear in the question
(D) Experts who give testimony on scientific or technical issues tend to hedge their conclusions by discussing the possibility of error.-----------------means discuss can reduce the risk to make unfair conclusion, means exclude some experts will not affect the fair conclusion. SUPPORT
(E) Expert witnesses in specialized fields often command fees that are so high that many people involved in litigation cannot afford their services.--------------fees are not discussed in the question, is not the question we have to consider. irrelevant

2. 17'
P: catfish swim erratically before earthquakes
C: proposed monitoring catfish to predict dangerous earthquakes.
Undermines the usefulness of the proposal: catfish are not very good sign to predict earthquakes
答案:E
(A) In Japan, which is subject to frequent earthquakes, the behavior of catfish has long been associated with earthquakes.----------prove that the catfish is useful. SUPPORT
(B) Mechanical methods for detecting earthquakes have not proved effective.---------------other methods are not discussed.
(C) Tremors lead to the release of hydrogen sulfide gas into water, thereby causing various fish and shellfish to behave erratically.--------------explain the reason why catfish are useful. irrelevant
(D) Careful construction can reduce the dangers posed by earthquakes.---------explain the importance of construction, but not the importance of forecast.
(E) Even very slight, fleeting tremors cause catfish to swim erratically.------------means catfish forecasting is not useful because slight fleeting tremors will not lead to earthquake. thus it is useless to alert people about those slight tremors. CORRECT

3. 26'
P: developers provide facilities, city cost nothing,
C: attract new citizens with new housing and new facilities
Flaw in plan: facilities will not attract citizens
答案:A
(A) Developers would pass along their costs to the buyer, thereby raising the cost of housing units beyond the ability of likely purchasers to afford them.-----------CORRECT. means facilities will not attract citizens because of the high price
(B) Light, nonpolluting industries have located in the area, offering more jobs and better-paying jobs than do the more-established industries in the area.------------Support the plan
(C) Other towns and cities nearby have yet to embark on any comparable plans to attract new citizens.----------Support the plan to attract citizens
(D) Most developers see the extra expense of providing municipal facilities as simply one of the many costs of doing business.-------------what developers see is not related to where citizens will be attracted
(E) Studies show that purchasers of new houses, especially first-time buyers, rank recreational resources as an important factor in deciding to buy a particular house.--------Support the plan

4. 22'
BG: issue coupons to get consumers and hope consumers to become loyalty.
P: HI should issue cents-off coupons
C:  attracting a large segment of potential consumers as loyal customers
Doubt the strategy: coupons are not guarantee to generate loyalty customers
答案: B
(A) Many consumers are unlikely to try new brands of products unless offered an inducement to do so.-------"many" here is too vague to be justify; inducement here means the coupons. this statement just state when customers can be attracted to try new brands, but nothing about the loyalty
(B) The consumers whose purchases are strongly influenced by cents-off coupons tend not to become loyal customers of any particular brand.-----------means coupons cannot guarantee the loyalty.CORRECT
(C) Many grocery stores attract customers by doubling the face value of manufacturer's coupons.--------------this just the action that many stores uses. the outcome is not know, so it cannot conclude sufficient outcome.
(D) Typically less than one-third of the coupons issued by a manufacturer are redeemed by consumers.------------means coupons are not effective used by customers, but does not mention the loyalty. if some customers use coupons and become loyalty customer, the plan is still  useful.
(E) A marketing campaign that uses cents-off coupons is most effective when combined with a television advertising campaign.-------------means tv ad campaign should be added. this is not discussed in the question.

5. 17'
P: our state has spent more money per mile on road improvements than any other state.
C: we have adequate road system
Undermine: spend more≠better
答案:E
(A) In the spokesperson's state, spending on road improvements has been increasing more slowly over the past six years than it has in several other states.---------what spending changes do not affect the situation.we cannot conclude the current situation about road system
(B) Adequacy of a state's road system is generally less important to a businessperson considering doing business there than is the availability of qualified employees.---------important to businessperson is not discussed in the question
(C) Over the past six years, numerous businesses have left the spokesperson's state, but about as many businesses have moved into the state.---------businesses move out and move in do not wholly reflect the rode system
(D) In general, the number of miles of road in a state's road system depends on both the area and the population of the state.------------the number of miles is not discussed in the question. we just focus on the quality
(E) Only states with seriously inadequate road systems need to spend large amounts of money on road improvements.--------------means more spend reflect the poor condition.CORRECT

作者: meckyona    时间: 2013-9-29 00:11
.People who have specialized knowledge about a
scientific or technical issue are systematically
excluded from juries for trials where the issue is
relevant. Thus, trial by jury is not a fair means of
settling disputes involving such issues.
Which one of the following, if true, most seriously
weakens the argument?

A.        This strengthen the conclusion
B.        This weaken the conclusion,right
C.        Irrelevant
D.        Possibilities are irrelevant
E.        Costs have nothing to do with fair

To reduce the danger to life and property posed by major earthquakes, scientists have been investigating several techniques for giving advance warning of dangerous earthquakes.  Since catfish swim erratically before earthquakes, some investigators have proposed monitoring catfish to predict dangerous earthquakes.
Which of the following, if true, most seriously undermines the usefulness of the proposal?
A this is strenghthen
B
C 原理,irrelevant or strenghthen
D irrelevant
E serious weaken, right

30
A
B irrelevant
C other cities’ plans are irrelevant
D
E irrelevant
这题在A D中摇摆…有点没懂,还是选A,如果房产价格高于支付者能承受的价格,那么不能吸引新移民

31
A there was an inducement
B right
C no
D irrelevant?
E tv is irrelevant

32
A
B 无关
C 无关
D no weaken
E right

OMG难得的全对QWQ渐渐摸到了门道,开心

作者: 览物之情    时间: 2013-10-9 18:32
10月9号
1 23s 一
Premise:People who have specialized knowledge about a
scientific or technical issue are systematically
excluded from juries for trials where the issue is
relevant.
Conclusion: Thus, trial by jury is not a fair means of
settling disputes involving such issues.
因果推理:有专业知识的人会被排除作为相关案件的陪审团成员,因此陪审团不是一个公平的解决相关问题的方式。
Which one of the following, if true, most seriously weakens the argument?
P because people are exclude so it’s fair to judge from a general view point.
(A) The more complicated the issue being litigated,the less likely it is that a juror without
specialized knowledge of the field involved will be able to comprehend the testimony
being given.------- It's not about how complicated the issue is, it's about fair.
B
(B) The more a juror knows about a particular scientific or technical issue involved in a
trial, the more likely it is that the juror will be prejudiced in favor of one of the litigating
parties before the trial begins.--------correct, this is about fair.
(C) Appointing an impartial 公正的arbitrator仲裁人 is not a fair means of settling disputes involving scientific or technical issues, because arbitrators tend to favor settlements in which both parties compromise on the issues.----------support the argument
(D) Experts who give testimony on scientific or technical issues tend to hedge their
conclusions by discussing the possibility of error.---------无关trail
(E) Expert witnesses in specialized fields often command fees that are so high that many
people involved in litigation cannot afford their services.--------it’s about be a jury not provide service.


2 18s
Background : To reduce the danger to life and property posed by major earthquakes, scientists have been investigating several techniques for giving advance warning of dangerous earthquakes.
Premise: Since catfish swim erratically before earthquakes
Conclusion:some investigators have proposed monitoring catfish to predict dangerous earthquakes.
Which of the following, if true, most seriously undermines the usefulness of the proposal?
方案评估:有效性,可行性,副作用 E
P: fish may swim erratically due to other factors.
A) In Japan, which is subject to frequent earthquakes, the behavior of catfish has long been associated with earthquakes.---------strength
(B) Mechanical methods for detecting earthquakes have not proved effective.-----strength
(C) Tremors lead to the release of hydrogen sulfide gas into water, thereby causing various fish and shellfish to behave erratically.-----background irrelevant
(D) Careful construction can reduce the dangers posed by earthquakes.---not about fish irrelevant
(E) Even very slight, fleeting tremors cause catfish to swim erratically.------correct.

3 23s
Background: A city plans to attract new citizens with new housing and new facilities such as parks, recreation centers, and libraries.
Plan: One component of the city's plan is to require that developers seeking permission to build this new housing provide these additional facilities at no cost to the city.
Which of the following, if true, would point to a possible flaw in the city's plan?
P : if no cost to the city, there will be cost to new citiszens and increase the housing price.
A
(A)        Developers would pass along their costs to the buyer, thereby raising the cost of housing units beyond the ability of likely purchasers to afford them.-------correct
(B) Light, nonpolluting industries have located in the area, offering more jobs and better-paying jobs than do the more-established industries in the area.-----irrelevant
(C) Other towns and cities nearby have yet to embark on any comparable plans to attract new citizens.--------irrelevant
(D) Most developers see the extra expense of providing municipal facilities as simply one of the many costs of doing business.----------not weaken
(E) Studies show that purchasers of new houses, especially first-time buyers, rank recreational resources as an important factor in deciding to buy a particular house.-----not weaken

4 26s
Premise: Manufacturers issue cents-off coupons to get consumers to try their brand of product with the hope that the consumers who try their brand will switch their brand loyalty.  
Conclusion: So in the initial marketing of their new brand X, Hartman Industries should issue cents-off coupons, thereby attracting a large segment of potential consumers as loyal customers.
因果推理:因果联系 people are less attracted by cents-off coupons B
Which of the following, if true, casts the most serious doubt on the likelihood that the marketing strategy recommended above will have the result that is claimed?
) Many consumers are unlikely to try new brands of products unless offered an inducement to do so.------not about the plan so irrelevant
(B) The consumers whose purchases are strongly influenced by cents-off coupons tend not to become loyal customers of any particular brand.------correct
(C) Many grocery stores attract customers by doubling the face value of manufacturer's coupons.-------not about this plan so irrelevant
(D) Typically less than one-third of the coupons issued by a manufacturer are redeemed by consumers.--------irrelevant
(E) A marketing campaign that uses cents-off coupons is most effective when combined with a television advertising campaign.-----not about loyal.

5 23s
Conclusion: Many businesspeople who have not been to our state believe that we have an inadequate road system.  Those people are mistaken,
Premise: as is obvious from the fact that in each of the past six years, our state has spent more money per mile on road improvements than any other state.
Which of the following, if true, most seriously undermines the reasoning in the spokesperson's argument? E
P: 因果推理因果联系 input money does not necessary make the road system good.
(A) In the spokesperson's state, spending on road improvements has been increasing more slowly over the past six years than it has in several other states.
(B) Adequacy of a state's road system is generally less important to a businessperson considering doing business there than is the availability of qualified employees.----irrelevant
(C) Over the past six years, numerous businesses have left the spokesperson's state, but about as many businesses have moved into the state.------irrelevant about the road system
(D) In general, the number of miles of road in a state's road system depends on both the area and the population of the state.-----irrelevant not about the road system
(E) Only states with seriously inadequate road systems need to spend large amounts of money on road improvements.------correct.这题和OG商场价格那题很像。

作者: w.melhere    时间: 2013-10-21 19:54
  1. 23‘  一开始没有看懂'People who have specialized knowledge about a scientific or technical issue are systematically excluded from juries for trials where the issue is relevant.' 这句。做的时候大概用技巧做了下,居然对了~  现在看了NN们的解释终于懂啦!
2. 19’
3. 21‘
4. 28’
5. 18‘
几天做的不错,虽然有几道都是做过的题了~  明天继续!
作者: cecidefok    时间: 2013-10-30 18:51
精练题
P:当审判团有具有与案件相关的专业知识的人时,这些人会被排除在外
C:审判团不是一种公平的方式
问削弱
推测:将有相关知识的人排除在外是公平的,防止了不公平意见的产生
A.说明了专业知识的重要性,没削弱
B.有专业知识的人可能会prejudge,这样会不公平...所以这方面上,审判团是公平的,削弱!!
C.提到arbitrator,不相关
D.提到expert不相关
E.与价格无关                选B

29.
P:某种鱼在地震前会不规律地游泳
C:研究人员认为可以用这种鱼来预测地震
问削弱
推测:这种鱼只在弱地震的时候才会不规律游泳..
选E

30.
fact:某市欲通过新住房和新建设施(如公园/娱乐中心/图书馆)来吸引新市民
   此市要求想获得建筑许可的developer不能对city产生cost
问flaw
推测:developer很多是new citizen,没有profit,吸引不了他们
选A

31.
P:生产商会在新出产品进入市场的时候印发优惠券来吸引消费者使用----从而改变他们的loyalty
C:HI退出产品X的时候也应该印发优惠券---从而吸引大量潜在顾客成为loyal customer
问削弱
推测:HI不适合印发优惠券
选B

32.
P:很多没来过我们州的商人认为我们州的道路系统差.其实他们错了,因为我们州每年都会投入比上一面要多的前在道路系统,每公里投入的钱会高于上一年
问削弱
推测:投入钱多,不代表就足够,可能只是修补以前得
选E

作者: yuehuasunday    时间: 2013-11-7 14:27

1.

Background: People who have specialized knowledge about a scientific or technical are systematically excluded

Premise: Excluded from juries for trials where the issue is relevant

Conclusion: Thus, trial by jury by jury is not a fair means of setting disputes

(A) Incorrect
(B) Correct
© Unrelated
(D) Unrelated
(E) Unrelated

2.

Background: Reduce the danger from the earthquakes

Premise: Catfish swim erratically before earthquakes

Conclusion: Investigators have proposed monitoring catfish to predict dangerious earthquakes

(E) Correct

3.

Background: City plans to attract new citizens with new housing and new facilities

Premise: Require developers seeking permission

Conclusion: At no cost to the city

(A) Correct

4.

Background: Manufacturers issue cents-off coupons to get consumers to try their brand of product with the hope that the consumers who try their brand will switch their brand loyalty.

Premise: So in the initial marketing of their new brand X, Hartman Industries should issue cents-off coupons,

Conclusion: Attracting a large segment of potential consumers as loyal customers

(B) Correct

5.

Background: State spokesperson:  Many businesspeople who have not been to our state believe that we have an inadequate road system.

Premise: Those people are mistaken

Conclusion: We have spent more money per mile on road improvements than any other state

(E) Correct






作者: yuehuasunday    时间: 2013-11-7 14:30

1.

Background: People who have specialized knowledge about a scientific or technical are systematically excluded

Premise: Excluded from juries for trials where the issue is relevant

Conclusion: Thus, trial by jury by jury is not a fair means of setting disputes

(A) Incorrect
(B) Correct
© Unrelated
(D) Unrelated
(E) Unrelated

2.

Background: Reduce the danger from the earthquakes

Premise: Catfish swim erratically before earthquakes

Conclusion: Investigators have proposed monitoring catfish to predict dangerious earthquakes

(E) Correct

3.

Background: City plans to attract new citizens with new housing and new facilities

Premise: Require developers seeking permission

Conclusion: At no cost to the city

(A) Correct

4.

Background: Manufacturers issue cents-off coupons to get consumers to try their brand of product with the hope that the consumers who try their brand will switch their brand loyalty.

Premise: So in the initial marketing of their new brand X, Hartman Industries should issue cents-off coupons,

Conclusion: Attracting a large segment of potential consumers as loyal customers

(B) Correct

5.

Background: State spokesperson:  Many businesspeople who have not been to our state believe that we have an inadequate road system.

Premise: Those people are mistaken

Conclusion: We have spent more money per mile on road improvements than any other state

(E) Correct






作者: irenetopia    时间: 2013-11-21 14:11
【精练】 1:45
1.People who have specialized knowledge about a
scientific or technical issue are systematically
excluded from juries for trials where the issue is
relevant. Thus, trial by jury is not a fair means of
settling disputes involving such issues.
Which one of the following, if true, most seriously
weakens the argument?
Weaken Question Problem Set  NO.9

逻辑链:排除与issue有关的scientific或technical的knowledge会造成一个不公平的means
问削弱
思路:就是说排除K不会造成不公平的M或者不排除K,M更不公平

(A) The more complicated the issue being litigated,
the less likely it is that a juror without
specialized knowledge of the field involved
will be able to comprehend the testimony
being given.——因果倒置,排除
(B) The more a juror knows about a particular
scientific or technical issue involved in a
trial, the more likely it is that the juror will be
prejudiced in favor of one of the litigating
parties before the trial begins.——bingo!囊括K的M更主观了!
(C) Appointing an impartial arbitrator is not a fair
means of settling disputes involving
scientific or technical issues, because
arbitrators tend to favor settlements in which
both parties compromise on the issues.——话题转移说appointing an impartial arbitrator是不公平的,排除
(D) Experts who give testimony on scientific or
technical issues tend to hedge their
conclusions by discussing the possibility of
error.——算是support
(E) Expert witnesses in specialized fields often
command fees that are so high that many
people involved in litigation cannot afford
their services.——话题转移,排除


【逻辑链】
29. 1:20
To reduce the danger to life and property posed by major earthquakes, scientists have been investigating several techniques for giving advance warning of dangerous earthquakes.  Since catfish swim erratically before earthquakes, some investigators have proposed monitoring catfish to predict dangerous earthquakes.
Which of the following, if true, most seriously undermines the usefulness of the proposal?

逻辑链:为了reduce earthquake的损失,专家就想办法预测地震
发现catfish在地震前会不规律的游动,所以一些人认为catfish可以预测地震
问削弱
思路:就是找catfish不一定就是在地震前swim erratically

(A) In Japan, which is subject to frequent earthquakes, the behavior of catfish has long been associated with earthquakes.——support
(B) Mechanical methods for detecting earthquakes have not proved effective.——irrelevant
(C) Tremors lead to the release of hydrogen sulfide gas into water, thereby causing various fish and shellfish to behave erratically.——说的是震荡带来的属性,irrelevant
(D) Careful construction can reduce the dangers posed by earthquakes.——irrelevant
(E) Even very slight, fleeting tremors cause catfish to swim erratically.——bingo!因为 fleeting tremors就能让catfish乱游,所以会影响预测准确

30. 1:36
A city plans to attract new citizens with new housing and new facilities such as parks, recreation centers, and libraries.  One component of the city's plan is to require that developers seeking permission to build this new housing provide these additional facilities at no cost to the city.
Which of the following, if true, would point to a possible flaw in the city's plan?

逻辑链:A想要用新的房子和设施吸引新的citizens
其中一个组成部门是希望这些new housing提供additional facilities with no cost
问flaw

思路:选项包含题目所有stimulus,没有新的information,可能是no cost,也可能是新的facilities并不受欢迎。不管是什么结果,判断计划,最终影响的是“attract new citizens”

(A) Developers would pass along their costs to the buyer, thereby raising the cost of housing units beyond the ability of likely purchasers to afford them.——bingo!成本增加,倒是new citizens的购买力,最终影响结果
(B) Light, nonpolluting industries have located in the area, offering more jobs and better-paying jobs than do the more-established industries in the area.——support
(C) Other towns and cities nearby have yet to embark on any comparable plans to attract new citizens.——new information
(D) Most developers see the extra expense of providing municipal facilities as simply one of the many costs of doing business.——support
(E) Studies show that purchasers of new houses, especially first-time buyers, rank recreational resources as an important factor in deciding to buy a particular house.——support

31. 1:10
Manufacturers issue cents-off coupons to get consumers to try their brand of product with the hope that the consumers who try their brand will switch their brand loyalty.  So in the initial marketing of their new brand X, Hartman Industries should issue cents-off coupons, thereby attracting a large segment of potential consumers as loyal customers.
Which of the following, if true, casts the most serious doubt on the likelihood that the marketing strategy recommended above will have the result that is claimed?
逻辑链:为了吸引更多地客户变成loyalty customers,厂商在推出新产品X时发型了折价券,因此可以吸引大批购买者成为loyalty customers。
问削弱:
思路:也许customers只是折价券的爱好者,选项包含,折价券并不能产生loyalty customers的因素

(A) Many consumers are unlikely to try new brands of products unless offered an inducement to do so.——其实这个无关,有无折价券对这个没有影响。现在讨论的是有关于折价卷strategy的问题
(B) The consumers whose purchases are strongly influenced by cents-off coupons tend not to become loyal customers of any particular brand.——bingo!人只是为券吸引,并不真正对brand感兴趣,就不会成为loyalty customers
(C) Many grocery stores attract customers by doubling the face value of manufacturer's coupons.——算是support了
(D) Typically less than one-third of the coupons issued by a manufacturer are redeemed by consumers.——无关
(E) A marketing campaign that uses cents-off coupons is most effective when combined with a television advertising campaign.——扯太远了!

32. 1:30
State spokesperson:  Many businesspeople who have not been to our state believe that we have an inadequate road system.  Those people are mistaken, as is obvious from the fact that in each of the past six years, our state has spent more money per mile on road improvements than any other state.
Which of the following, if true, most seriously undermines the reasoning in the spokesperson's argument?
逻辑链:一些不来我们state做生意的商人认为我们路况不好
其实不是酱紫的因为我们过去六年,每年都花很多钱在路上
问削弱
思路:因为路况不好才修路啊

(A) In the spokesperson's state, spending on road improvements has been increasing more slowly over the past six years than it has in several other states.——修的慢所以人家不来?扯
(B) Adequacy of a state's road system is generally less important to a businessperson considering doing business there than is the availability of qualified employees.——质疑了前提,排除
(C) Over the past six years, numerous businesses have left the spokesperson's state, but about as many businesses have moved into the state.——说到了商人的状态,与原因无关
(D) In general, the number of miles of road in a state's road system depends on both the area and the population of the state.——仅说道路况的转台,无关
(E) Only states with seriously inadequate road systems need to spend large amounts of money on road improvements.——bingo!因为路不好,所以修嘛
作者: goodluck22    时间: 2014-3-3 21:57
Premise: people who have specialized knowledge was excluded from juries for trials where the issue is relevant
Conclusion: is not a fair means
Prephrase: it is necessary because people will be affected
A.        support
B.        cORRECT
C.        irrelevant
D.        experts irrelevant
E.        fee is not the case here

Background: to reduce the danger of life and property by major earthquake, scientist have been investigating several technique to advance warning
Premise: catfish swim erratically before earthquake
Conclusion: monitor catfish to predict
Prephrase: other factor will cause catfish to swim , therefore catfish is not  the
Case
A.SUPPORT
B.SUPPORT
C. SUPPORT
D. background
E. correct

Premise: city plan is to require developers to build the new housing  provide addition facilities free charge.
Conclusion : to attract new citizens with new housing and facilities
Prephrase: will not attract the new citizens/ buyer will paid more money
A.        correct
B.        Irrelevant
C.        Out of scope
D.        Support
E.        Background

Premise: manufacturers issue coupons to attract consumer to be loal
Conclusion: to attract consumer-> brand x should issue coupons
Prephrase: competitors will use same methoud/ will not become loya
A.        support
B.        correct
C.        background out of scope
D.        irrelevant
E.        background

Premise: many businesspeople who have not been to our states believe that we have inadequate road . However, more money per mile on road improvement than other states
Conclusion: those people are mistaken
Prephrase: because the road on our states was very inadequate so we have to spend more money.
A.        irrelevant
B.        irrelevant
C.        irrelevant
D.        irrelevant
E.        Correct

作者: cyndichiang    时间: 2014-4-7 14:18
精炼:
1.36‘’
Premise: People who have specialized knowledge about a scientific or technical issue are systematically
excluded from juries for trials where the issue is relevant.
Conclusion:trial by jury is not a fair means of settling disputes involving such issues.
Q:Weaken
prephrase: 证明specialized knowledge about a scientific or technical issue 是fair必不可少的(错)
(A) The more complicated the issue being litigated,
the less likely it is that a juror without
specialized knowledge of the field involved
will be able to comprehend the testimony
being given.----选错了,选了A,--strengthen,选项意思是如果那些有专业知识的人被排除出陪审团,那么陪审员就更难理解一些证据,因此加强了这是一个unfair的trial
(B) The more a juror knows about a particular
scientific or technical issue involved in a
trial, the more likely it is that the juror will be
prejudiced in favor of one of the litigating
parties before the trial begins.----Correct,正好与结论相反,指出了科技知识知道越多越有偏见,那说明应该把有这些知识的人排除在外,才能不偏见也就是:excluded→fair与结论相反,有因无果,削弱
(C) Appointing an impartial arbitrator is not a fair
means of settling disputes involving
scientific or technical issues, because
arbitrators tend to favor settlements in which
both parties compromise on the issues.-----irrelevant;说另一种方法unfair对于这种方法是否能削弱结论的程度没有影响
(D) Experts who give testimony on scientific or
technical issues tend to hedge their
conclusions by discussing the possibility of
error.----irrelevant;文章并没有说回避错误的可能性对公平与否的程度产生什么作用
(E) Expert witnesses in specialized fields often
command fees that are so high that many
people involved in litigation cannot afford
their services.-----irrelevant,文章也没提到费用


逻辑链:
2. 17‘’
Premsie: catfish swim erratically before earthquakes
Conclusion: monitoring catfish is a way to predict dangerous earthquakes.
Q:weaken
prephrase: catish swim erratically when there is no earthquakes but just a small interfere


3.37''
premise: developers seeking permission to build this new housing provide these additional facilities at no cost to the city.
Conclusion: this plan can attract new citizens
Q:weaken
prephrase: new citizens burden these cost of new housing and facilities


4.37''
premise: cent-off campaign can lead customers to switch their brand loyalties.
Conclusion:cents-off coupons of X can  attract a large segment of potential consumers as loyal customers
Q:weaken;
prephrase: these customers who change their brand loyalties cannot be loyal customers.


5.22''
premise:in each of the past six years, our state has spent more money per mile on road improvements than any other state.
Conclusion: That our state has inadequate road system is wrong.
Q:weaken
Prephrase: even though the state has more improvements ,its road system is still poor and left behind any other states.

作者: simonleo    时间: 2014-5-1 12:06
BEABE
1.
Time:45s
P:people who have specialized knowledge about a scientific or technical issue are excluded from juries for trials where the issue is relevant.
C:trial by jury is not a fair means of settling disputes involving such issues.
Pre:削弱。说明并不存在不fair即可
(A) The more complicated the issue being litigated,
the less likely it is that a juror without
specialized knowledge of the field involved
will be able to comprehend the testimony
being given. 说问题复杂程度,那如果不那么复杂,juror可以理解,不trial则是不公平的啊,juror能不能理解是不无关的吧
(B) The more a juror knows about a particular
scientific or technical issue involved in a
trial, the more likely it is that the juror will be
prejudiced in favor of one of the litigating
parties before the trial begins. 正解,说明讲这类juror排除是很合理、很公平的。
(C) Appointing an impartial arbitrator is not a fair
means of settling disputes involving
scientific or technical issues, because
arbitrators tend to favor settlements in which
both parties compromise on the issues.无关
(D) Experts who give testimony on scientific or
technical issues tend to hedge their
conclusions by discussing the possibility of
error. 加强了,这样的话没有专业知识则很容易被误导,则这类trial的处理上不是很fair
(E) Expert witnesses in specialized fields often
command fees that are so high that many
people involved in litigation cannot afford
their services. 无关,与费用无关。

最开始的时候题意理解的不是很清晰

2.
Time:39s
Background: earthquick cause damage to life and property, scientists investigate techniques for giving advance warning of dangerous earthquakes.
P:catfish swim erratically before earthquakes
C:monitor catfish to predict dangerous earthquakes.
Pre:要保证catfish在其他情况下不会swim erratically啊
(A) In Japan, which is subject to frequent earthquakes, the behavior of catfish has long been associated with earthquakes. 不能削弱,加强
(B) Mechanical methods for detecting earthquakes have not proved effective. 无关
(C) Tremors lead to the release of hydrogen sulfide gas into water, thereby causing various fish and shellfish to behave erratically. 不能削弱,即使很多鱼都会反常,用catfish来预测也不能说是不合理的。
(D) Careful construction can reduce the dangers posed by earthquakes. 无关
(E) Even very slight, fleeting tremors cause catfish to swim erratically. 正解, 说明了即使是很轻的震动,catfish都会有反常表现,即它不是一个很好的预测dangerous earthquakes的指示物。

3.
Time:50s
P:A city plans to attract new citizens with new housing and new facilities
One component of the city's plan is to require that developers build this new housing provide these additional facilities at no cost to the city.
Pre:削弱plan。指出flaw in the plan,即让plan不能达到预想效果。
(A) Developers would pass along their costs to the buyer, thereby raising the cost of housing units beyond the ability of likely purchasers to afford them. 正解,可以指出plan的flaw,plan 要求no cost to the city, 则cost to developer就会高,则卖给消费者价格高,消费者就不会来买,则plan没有起到attract的作用。
(B) Light, nonpolluting industries have located in the area, offering more jobs and better-paying jobs than do the more-established industries in the area.虽然这个可以attract新居民,但不能说明the plan不能attract
(C) Other towns and cities nearby have yet to embark on any comparable plans to attract new citizens.其他城市用了不代表我用了就不能attract
(D) Most developers see the extra expense of providing municipal facilities as simply one of the many costs of doing business. 可以加强,因为说developer是愿意不将cost推给城市的,则plan可以很好的实施进行。
(E) Studies show that purchasers of new houses, especially first-time buyers, rank recreational resources as an important factor in deciding to buy a particular house.加强,说明plan可以达到臆想中结果。

该题有个关键点,即题干中没有明确的结论,从问句中的flaw in the city's plan要可以推出,要求我们找到flaw,其结果可以让plan不能达到预想中的结果,即可以削弱plan的效果。

4.
Time:1:04
P:Manufacturers issue cents-off coupons to get consumers to try their brand of product with the hope that the consumers who try their brand will switch their brand loyalty.
C:Hartman Industries should issue cents-off coupons, thereby attracting a large segment of potential consumers as loyal customers.
Pre:削弱。吸引来了不一定就可以成为loyal customers。前提也只是with the hope
(A) Many consumers are unlikely to try new brands of products unless offered an inducement to do so. 不能削弱,反而说明了有inducement所以可以可以吸引到很多顾客。
(B) The consumers whose purchases are strongly influenced by cents-off coupons tend not to become loyal customers of any particular brand. countender正解,可以说明不会成为loyalty
(C) Many grocery stores attract customers by doubling the face value of manufacturer's coupons.无关
(D) Typically less than one-third of the coupons issued by a manufacturer are redeemed by consumers.不能削弱,即使很少一部分redeem但是只要redeem了,即他们是potential consumers,就有可能变成loyalty
(E) A marketing campaign that uses cents-off coupons is most effective when combined with a television advertising campaign.无关。

5.
Time:38s
P:we spend more money per mile on road improvements than any other state.
C:so the believe that we have an inadequate road system is mistaken.
Pre:花钱多不代表道路完善。也许基础比别人差,你花了更多钱你的道路系统仍然可能不完善。
(A) In the spokesperson's state, spending on road improvements has been increasing more slowly over the past six years than it has in several other states.花费的增长慢不说明问题,我花费多按照spokeman的思维就是会比别人的道路完善。
(B) Adequacy of a state's road system is generally less important to a businessperson considering doing business there than is the availability of qualified employees.无关
(C) Over the past six years, numerous businesses have left the spokesperson's state, but about as many businesses have moved into the state.无关
(D) In general, the number of miles of road in a state's road system depends on both the area and the population of the state.contender,说明了即使你花费多,你的mile数量任然是不够的,因为要考虑population。但是不好的一点是,我们并不知道population的数量是大是小。
(E) Only states with seriously inadequate road systems need to spend large amounts of money on road improvements. 正解,你花费多正好说明了你inadequate。

作者: 采药去    时间: 2014-10-24 21:13
精练题--A,没看懂好多生词
[color=Red]exdude ɪɡ'zud]
vt. 散发;流出;使渗出
vi. 流出;渗出;发散

litigated ['lɪtɪɡet] vt. 在法庭相争;提出诉讼;vi. 对簿法庭
condusion 结论


p:某领域的专家从陪审团流出,当issue 相关的时候
C:陪审团的审判是不公平的
问weaken

A.越复杂的问题被提出诉讼,没有专家的陪审团听懂的证词的可能性更小--无关
B.陪审人对专业的问题知道的越多,越有可能在审判之前就歧视其中一个诉讼方--正确
C.任命一个公平的仲裁者是unfair的--无关
D.support
E.费用高--无关


错了,正确答案选B
逻辑链
E
A
B
E

作者: 夏日MUSE    时间: 2014-10-25 17:06
精炼B
29  E
30  A
31  B
32  E
作者: spencerchan    时间: 2015-8-13 14:48
43’’
P: People who have specialized knowledge about a scientific or technical issue are excluded from juries for trials where issue is relevant
C: trial by jury is not a fair means of setting disputes involving such issues
Prep: trail by jury is a fair means of setting such issues
A: CORRECT
B: Strengthen
C: Strengthen
D: Strengthen
E: Strengthen

21’’
P: Catfish swim erratically before earthquake
C: Scientists predict earthquake by observing catfish
Prep: Catfish swim erratically in many situations/ Time
C: CORRECT
A: Strengthen
B: Strengthen
D: Irrelevant
E: Strengthen

43’’
P: Build new facilities
C: By seeking developers to build it at no cost to the city
Prep: Seeking developers needs money; I really do not understand the question…. Just guess the logic link
A: CORRECT
B: Irrelevant
C: Irrelevant
D: Irrelevant
E: Irrelevant

29’’
P: New brand X issues cents-off coupons
C: It would attract customers who would become loyal customer
Prep: The product X has bad quality, so customers wont buy it again
B: CORRECT
A: Not weaken or strengthen
C: Irrelevant
D: Irrelevant/ slight strengthen
E: Irrelevant

23’’
P: Our state has spent more money in road improvement than others
C: Our road condition is better
Prep: More money=Bad road condition
E: CORRECT
A: Not strengthen or weaken
B: Irrelevant
C: Irrelevant
D: Irrelevant to number of road

做的时候明明觉得很简单。。还是错了两道,好好分析!
作者: 千晤meli    时间: 2015-8-18 12:40
精炼题完全没看懂题目嘛!
重看了好几遍...Thus, trial by jury is not a fair means of
settling disputes involving such issues. 的意思原来是TbyJ 不是一个公正的方法。
那么消弱就要说排除specialized tech.的人有什么好处咯。
作者: MIA926    时间: 2015-9-1 08:24
20150901 CR小分队
【精练】
1.People who have specialized knowledge about a scientific or technical issue are systematically excluded from juries for trials where the issue is relevant. Thus, trial by jury is not a fair means of settling disputes involving such issues.
Which one of the following, if true, most seriously weakens the argument?
计时: 19.30s
逻辑链:jury会排除有专业知识的人参加相关的issue 因此trial by 这些jury是不公平的
推测:有专业知识的人参加可能会用专业知识操纵事实更不公平
Weaken Question Problem Set  NO.9
(A) The more complicated the issue being litigated,
the less likely it is that a juror without
specialized knowledge of the field involved
will be able to comprehend the testimony
being given. 加强
(B) The more a juror knows about a particular
scientific or technical issue involved in a
trial, the more likely it is that the juror will be
prejudiced in favor of one of the litigating
parties before the trial begins. 正确
(C) Appointing an impartial arbitrator is not a fair
means of settling disputes involving
scientific or technical issues, because
arbitrators tend to favor settlements in which
both parties compromise on the issues. 无关
(D) Experts who give testimony on scientific or
technical issues tend to hedge their
conclusions by discussing the possibility of
error. 无关
(E) Expert witnesses in specialized fields often
command fees that are so high that many
people involved in litigation cannot afford
their services. 无关

【逻辑链】
29.
To reduce the danger to life and property posed by major earthquakes, scientists have been investigating several techniques for giving advance warning of dangerous earthquakes.  Since catfish swim erratically before earthquakes, some investigators have proposed monitoring catfish to predict dangerous earthquakes.
Which of the following, if true, most seriously undermines the usefulness of the proposal?
逻辑链:某种鱼地震前会很奇怪,科学家觉得可以通过观测这种鱼来预测地震
推测:可能有其他原因导致这种鱼也会变得不正常
(A) In Japan, which is subject to frequent earthquakes, the behavior of catfish has long been associated with earthquakes. 加强
(B) Mechanical methods for detecting earthquakes have not proved effective. 无关
(C) Tremors lead to the release of hydrogen sulfide gas into water, thereby causing various fish and shellfish to behave erratically. 加强
(D) Careful construction can reduce the dangers posed by earthquakes. 无关
(E) Even very slight, fleeting tremors cause catfish to swim erratically. 正确

30.
A city plans to attract new citizens with new housing and new facilities such as parks, recreation centers, and libraries.  One component of the city's plan is to require that developers seeking permission to build this new housing provide these additional facilities at no cost to the city.
计划建设施吸引居民,但是城市不给钱
方案题:不可行,不良后果
Which of the following, if true, would point to a possible flaw in the city's plan?
(A) Developers would pass along their costs to the buyer, thereby raising the cost of housing units beyond the ability of likely purchasers to afford them. 正确
(B) Light, nonpolluting industries have located in the area, offering more jobs and better-paying jobs than do the more-established industries in the area. 无关
(C) Other towns and cities nearby have yet to embark on any comparable plans to attract new citizens. 无关
(D) Most developers see the extra expense of providing municipal facilities as simply one of the many costs of doing business. 加强plan
(E) Studies show that purchasers of new houses, especially first-time buyers, rank recreational resources as an important factor in deciding to buy a particular house. 加强plan

31.
Manufacturers issue cents-off coupons to get consumers to try their brand of product with the hope that the consumers who try their brand will switch their brand loyalty.  So in the initial marketing of their new brand X, Hartman Industries should issue cents-off coupons, thereby attracting a large segment of potential consumers as loyal customers.
逻辑链:发某种债券来吸引忠实客户
推测:被这样吸引来的人很容易又被别的类似优惠债券吸引走了
Which of the following, if true, casts the most serious doubt on the likelihood that the marketing strategy recommended above will have the result that is claimed?
(A) Many consumers are unlikely to try new brands of products unless offered an inducement to do so. 加强
(B) The consumers whose purchases are strongly influenced by cents-off coupons tend not to become loyal customers of any particular brand. 正确
(C) Many grocery stores attract customers by doubling the face value of manufacturer's coupons. 无关
(D) Typically less than one-third of the coupons issued by a manufacturer are redeemed by consumers. 无关
(E) A marketing campaign that uses cents-off coupons is most effective when combined with a television advertising campaign. 无关

32.
State spokesperson:  Many businesspeople who have not been to our state believe that we have an inadequate road system.  Those people are mistaken, as is obvious from the fact that in each of the past six years, our state has spent more money per mile on road improvements than any other state.
逻辑链:没来过我们这里的人说我们的道路建设不足,但是他们错了,因为过去六年间我们比其他地方花了更多钱在道路建设上
推测:就是不足所以多建啊…
Which of the following, if true, most seriously undermines the reasoning in the spokesperson's argument?
(A) In the spokesperson's state, spending on road improvements has been increasing more slowly over the past six years than it has in several other states. 速度无关
(B) Adequacy of a state's road system is generally less important to a businessperson considering doing business there than is the availability of qualified employees. 无关 business
(C) Over the past six years, numerous businesses have left the spokesperson's state, but about as many businesses have moved into the state. 无关business
(D) In general, the number of miles of road in a state's road system depends on both the area and the population of the state. 无关
(E) Only states with seriously inadequate road systems need to spend large amounts of money on road improvements. 正确

作者: Geranium_Bayy    时间: 2016-3-26 23:06
【精练】
(做错)1.People who have specialized knowledge about a scientific or technical issue are systematically excluded from juries for trials where the issue is relevant. Thus, trial by jury is not a fair means of settling disputes involving such issues.
Which one of the following, if true, most seriously weakens the argument?
Background information: expert doesn’t need juries for trials where the issue is relevant.
Conclusion: juries for trials is not a fair method of settling disputes
Weaken: 1、experts have some secrets that doesn’t want to let juries know.
2、some evidences show that juries can give a impartial outcome.
(A) The more complicated the issue being litigated, the less likely it is that a juror without specialized knowledge of the field involved will be able to comprehend the testimony being given. irrelevant
(B) The more a juror knows about a particular scientific or technical issue involved in a trial, the more likely it is that the juror will be prejudiced in favor of one of the litigating parties before the trial begins.  irrelevant
(C) Appointing an impartial arbitrator is not a fair means of settling disputes involving scientific or technical issues, because arbitrators tend to favor settlements in which both parties compromise on the issues. Irrelevant
(D) Experts who give testimony on scientific or technical issues tend to hedge their conclusions by discussing the possibility of error. Correct
(E) Expert witnesses in specialized fields often command fees that are so high that many people involved in litigation cannot afford their services. We don’t talk about the fees

【逻辑链】
29.
To reduce the danger to life and property posed by major earthquakes, scientists have been investigating several techniques for giving advance warning of dangerous earthquakes.  Since catfish swim erratically before earthquakes, some investigators have proposed monitoring catfish to predict dangerous earthquakes.
Which of the following, if true, most seriously undermines the usefulness of the proposal?
Background: scientists want to protects people’s life and property. They want to investigate some advanced method to warn the earthquake.
Premise: before earthquake, catfish will have an erratic behaviors.
Conclusion: scientists can observe catfish to predict earthquake.
Weaken: 1、catfish’s erratic behaviors are just accidental. May be there are other factors result in this phenomenon.
2、some causes result in catfish’s leave or extinct
3、there are other better methods to predict earthquake ( not that frequent)
(A) In Japan, which is subject to frequent earthquakes, the behavior of catfish has long been associated with earthquakes. strengthen
(B) Mechanical methods for detecting earthquakes have not proved effective. Irrelevant, we don’t talk about mechanical methods
(C) Tremors lead to the release of hydrogen sulfide gas into water, thereby causing various fish and shellfish to behave erratically. We talk about catfish, not various fish and shellfish. They may include catfish or may not.
(D) Careful construction can reduce the dangers posed by earthquakes. irrelevant
(E) Even very slight, fleeting tremors cause catfish to swim erratically. Correct

30.
(做错)A city plans to attract new citizens with new housing and new facilities such as parks, recreation centers, and libraries.  One component of the city's plan is to require that developers seeking permission to build this new housing provide these additional facilities at no cost to the city.
Which of the following, if true, would point to a possible flaw in the city's plan?
Background information: new housing and new facility will attract new citizens
Premise: to find a permission that free of new facility.
  Weaken: 1、can’t find 2、even though new facility will not increase cost to this country, there are other objects that will increase the cost to the country.
(A) Developers would pass along their costs to the buyer, thereby raising the cost of housing units beyond the ability of likely purchasers to afford them. irrelevant
(B) Light, nonpolluting industries have located in the area, offering more jobs and better-paying jobs than do the more-established industries in the area. Irrelevant comparison
(C) Other towns and cities nearby have yet to embark on any comparable plans to attract new citizens. Irrelevant comparison
(D) Most developers see the extra expense of providing municipal facilities as simply one of the many costs of doing business.  Correct
(E) Studies show that purchasers of new houses, especially first-time buyers, rank recreational resources as an important factor in deciding to buy a particular house.  Irrelevant

31.
(做错)Manufacturers issue cents-off coupons to get consumers to try their brand of product with the hope that the consumers who try their brand will switch their brand loyalty.  So in the initial marketing of their new brand X, Hartman Industries should issue cents-off coupons, thereby attracting a large segment of potential consumers as loyal customers.
Which of the following, if true, casts the most serious doubt on the likelihood that the marketing strategy recommended above will have the result that is claimed?
Premise: send coupons can make the consumers loyal
Conclusion: coupons——loyal consumers——attract potential consumers
Weaken: 1、people those who bought this company’s product ever will become loyal because of coupons, but those who never bought the product ever will not be attracted by the coupons/ those who never bought products will not concern this company’s activity, like coupon
2、Coupons is only useful for old brand , not new introduced brand.
(A) Many consumers are unlikely to try new brands of products unless offered an inducement to do so. strengthen
(B) The consumers whose purchases are strongly influenced by cents-off coupons tend not to become loyal customers of any particular brand. strengthen
(C) Many grocery stores attract customers by doubling the face value of manufacturer's coupons. irrelevant
(D) Typically less than one-third of the coupons issued by a manufacturer are redeemed by consumers. correct
(E) A marketing campaign that uses cents-off coupons is most effective when combined with a television advertising campaign. irrelevant

32.
State spokesperson:  Many businesspeople who have not been to our state believe that we have an inadequate road system.  Those people are mistaken, as is obvious from the fact that in each of the past six years, our state has spent more money per mile on road improvements than any other state.
Which of the following, if true, most seriously undermines the reasoning in the spokesperson's argument?
Background: many businesses men think XXX’s road system is not good.
Premise: XXX spent more money on road improvement in each of six years than any other states.
Conclusion: these people are incorrect
Weaken: effective comparsion
(A) In the spokesperson's state, spending on road improvements has been increasing more slowly over the past six years than it has in several other states. Only if the total spending is more than other states is okay. Otherwise, the option proposes “several other states”
(B) Adequacy of a state's road system is generally less important to a businessperson considering doing business there than is the availability of qualified employees. irrelevant
(C) Over the past six years, numerous businesses have left the spokesperson's state, but about as many businesses have moved into the state.  irrelevant
(D) In general, the number of miles of road in a state's road system depends on both the area and the population of the state. irrelevant
(E) Only states with seriously inadequate road systems need to spend large amounts of money on road improvements. correct

作者: beabetterself    时间: 2016-3-29 07:56
请问如何加入CR小分队呢?直接在这里写出回答就可以了吗 谢谢!!
作者: fesche    时间: 2016-5-10 01:07
people know science and tech are excluded from trial if it is relevant
trial is not fair if it involves such issues
weaken
因:技术 科技
果:有这些技术的不用trial是不平等的
平等的一面,他因不平等,因果不相连
BE之间考虑,B,体现unfair性,E没有体现不平等
(A) The more complicated the issue being litigated, the less likely it is that a juror without specialized knowledge of the field involved will be able to comprehend the testimony being given.
(B) The more a juror knows about a particular scientific or technical issue involved in a trial, the more likely it is that the juror will be prejudiced in favor of one of the litigating
parties before the trial begins.
(C) Appointing an impartial arbitrator is not a fair means of settling disputes involving scientific or technical issues, because arbitrators tend to favor settlements in which both parties compromise on the issues.
(D) Experts who give testimony on scientific or technical issues tend to hedge their conclusions by discussing the possibility of error.
(E) Expert witnesses in specialized fields often command fees that are so high that many people involved in litigation cannot afford their services.

predict earthquake
fish erratically behavior
suggestion observing fish
weaken
fish is always erratically swimming
erratically swimming does not mean earth quake, can be other reason
where there is fish, no chance of earth quake
E
(A) In Japan, which is subject to frequent earthquakes, the behavior of catfish has long been associated with earthquakes.
(B) Mechanical methods for detecting earthquakes have not proved effective.
(C) Tremors lead to the release of hydrogen sulfide gas into water, thereby causing various fish and shellfish to behave erratically.
(D) Careful construction can reduce the dangers posed by earthquakes.
(E) Even very slight, fleeting tremors cause catfish to swim erratically.

city wants to build new parks etc
require developers seeking permission add no additional cost to the city
flaw, weaken
no cost no one can build the parks
a
(A) Developers would pass along their costs to the buyer, thereby raising the cost of housing units beyond the ability of likely purchasers to afford them.
(B) Light, nonpolluting industries have located in the area, offering more jobs and better-paying jobs than do the more-established industries in the area.
(C) Other towns and cities nearby have yet to embark on any comparable plans to attract new citizens.
(D) Most developers see the extra expense of providing municipal facilities as simply one of the many costs of doing business.
(E) Studies show that purchasers of new houses, especially first-time buyers, rank recreational resources as an important factor in deciding to buy a particular house.

give coupon to attract new customer, become a loyal one
there for, brand X should issue coupon to attract customers
因:issue coupon
果:attract customers and become a loyal one
weaken
won’t be loyal one, no one wants the product even it’s given cent off
B,C也有影响因素,但是没有最终讲明不能成为loyal customers
(A) Many consumers are unlikely to try new brands of products unless offered an inducement to do so.
(B) The consumers whose purchases are strongly influenced by cents-off coupons tend not to become loyal customers of any particular brand.
(C) Many grocery stores attract customers by doubling the face value of manufacturer's coupons.
(D) Typically less than one-third of the coupons issued by a manufacturer are redeemed by consumers.
(E) A marketing campaign that uses cents-off coupons is most effective when combined with a television advertising campaign.

spend more money per mile in road than other states
statement of inadequate road system is wrong
weaken
money was not spent in extending but maintaining
E
A) In the spokesperson's state, spending on road improvements has been increasing more slowly over the past six years than it has in several other states.
(B) Adequacy of a state's road system is generally less important to a businessperson considering doing business there than is the availability of qualified employees.
(C) Over the past six years, numerous businesses have left the spokesperson's state, but about as many businesses have moved into the state.
(D) In general, the number of miles of road in a state's road system depends on both the area and the population of the state.
(E) Only states with seriously inadequate road systems need to spend large amounts of money on road improvements.

作者: hedyail    时间: 2016-8-30 02:33
错1对4,感觉今天的题目挺简单的,都是weaken,错的那题因为词汇理解不当错了,加强单词量摄入

精炼
WEAKEN
看题 20s
PREMISE:people knows specifically technical issue are excluded from juries for trials
CON:trail by jury is not fair to settle disputes relating to these issues
GUESS:These people can only be exclued under rare circumstance
C  b

答案参考:
【精练】
1.People who have specialized knowledge about a
scientific or technical issue are systematically
excluded from juries for trials where the issue is
relevant. Thus, trial by jury is not a fair means of
settling disputes involving such issues.
Which one of the following, if true, most seriously
weakens the argument?
Weaken Question Problem Set  NO.9
(A) The more complicated the issue being litigated,
the less likely it is that a juror without
specialized knowledge of the field involved
will be able to comprehend the testimony
being given. - not related to juror
(B) The more a juror knows about a particular
scientific or technical issue involved in a
trial, the more likely it is that the juror will be
prejudiced in favor of one of the litigating
parties before the trial begins. - not related to juror
(C) Appointing an impartial arbitrator is not a fair
means of settling disputes involving
scientific or technical issues, because
arbitrators tend to favor settlements in which
both parties compromise on the issues.
(D) Experts who give testimony on scientific or
technical issues tend to hedge their
conclusions by discussing the possibility of
error. -think twice doesn't mean it is fair
(E) Expert witnesses in specialized fields often
command fees that are so high that many
people involved in litigation cannot afford
their services.  - not realted

逻辑链
2.21S WEAKEN
BG: People investigate way to advance warn earthquake.
PRE: Catfish swim erratically before earquake, it has a sign
CON: catfish can be used to monitor earthquake status.
E

3.20S WEAKEN
BG: City plans to build new facilities to attract new citizens. They plan there is no extra cost to the city.
A

4.28S WEAKEN
PRE: factories issue coupons to attract new customers then gain loyalty
CON: The brand X should try this method
B

5.18S WEAKEN
BG: people think the state has not enough road system
PRE: The state has spent a lot in the past 6 years
CON:Their thinking is wrong
E
作者: 弦了个弦    时间: 2016-9-5 14:22
精炼 People who have specialized knowledge about a
scientific or technical issue are systematically
excluded from juries for trials where the issue is
relevant.  C:Trial by jury is not a fair means of
settling disputes involving such issues.
(A) The more complicated the issue being litigated,
the less likely it is that a juror without
specialized knowledge of the field involved
will be able to comprehend the testimony
being given.  越复杂,越需要专家——unfair——strengthen
正确(B) The more a juror knows about a particular
scientific or technical issue involved in a
trial, the more likely it is that the juror will be
prejudiced in favor of one of the litigating
parties before the trial begins. 专家会有偏见,排除专家——排除有偏见的人——more fair—— weaken  CQ2 排除干扰因素
(C) Appointing an impartial arbitrator is not a fair
means of settling disputes involving
scientific or technical issues, because
arbitrators tend to favor settlements in which
both parties compromise on the issues. irrelevant
作者: 宋痂岛__    时间: 2017-12-10 22:30
D
E
A
B
E
精炼错了 会认真看的。
作者: May97    时间: 2018-1-21 08:55
T: 21s
P:People who have specialized knowledge about a scientific or technical issue are systematically excluded from juries for trials where the issue is relevant.
C:Thus, trial by jury is not a fair means of settling disputes involving such issues.
Weaken:这些有specialized knowledge的人可能会因为其他因素在评判时造成不公平
(A) The more complicated the issue being litigated, the less likely it is that a juror without specialized knowledge of the field involved will be able to comprehend the testimony being given.--->support
(B) The more a juror knows about a particular scientific or technical issue involved in a trial, the more likely it is that the juror will be prejudiced in favor of one of the litigating parties before the trial begins.--->correct
(C) Appointing an impartial arbitrator is not a fair means of settling disputes involving scientific or technical issues, because arbitrators tend to favor settlements in which both parties compromise on the issues.--->irreverent
(D) Experts who give testimony on scientific or technical issues tend to hedge their conclusions by discussing the possibility of error.--->irreverent
(E) Expert witnesses in specialized fields often command fees that are so high that many people involved in litigation cannot afford their services.

T: 21s
P: To reduce the danger to life and property posed by major earthquakes, scientists have been investigating several techniques for giving advance warning of dangerous earthquakes.  
C: Since catfish swim erratically before earthquakes, some investigators have proposed monitoring catfish to predict dangerous earthquakes.
Weaken: catfish swim erratically 不是地震发生前的典型特征
(A) In Japan, which is subject to frequent earthquakes, the behavior of catfish has long been associated with earthquakes.--->support
(B) Mechanical methods for detecting earthquakes have not proved effective.--->irreverent
(C) Tremors lead to the release of hydrogen sulfide gas into water, thereby causing various fish and shellfish to behave erratically.--->irreverent
(D) Careful construction can reduce the dangers posed by earthquakes.--->irreverent
(E) Even very slight, fleeting tremors cause catfish to swim erratically.--->correct

T: 25s
C: A city plans to attract new citizens with new housing and new facilities such as parks, recreation centers, and libraries.  
P: One component of the city's plan is to require that developers seeking permission to build this new housing provide these additional facilities at no cost to the city.
Flaw: 如果developers需要作出这样的承诺,他们可能不愿意在这座城市建房子
(A) Developers would pass along their costs to the buyer, thereby raising the cost of housing units beyond the ability of likely purchasers to afford them.--->correct
(B) Light, nonpolluting industries have located in the area, offering more jobs and better-paying jobs than do the more-established industries in the area.--->irreverent
(C) Other towns and cities nearby have yet to embark on any comparable plans to attract new citizens.--->irreverent
(D) Most developers see the extra expense of providing municipal facilities as simply one of the many costs of doing business.--->support
(E) Studies show that purchasers of new houses, especially first-time buyers, rank recreational resources as an important factor in deciding to buy a particular house.--->support

T: 26s
P:: Manufacturers issue cents-off coupons to get consumers to try their brand of product with the hope that the consumers who try their brand will switch their brand loyalty.  
C: So in the initial marketing of their new brand X, Hartman Industries should issue cents-off coupons, thereby attracting a large segment of potential consumers as loyal customers.
Weaken: 1) consumers who try their brand 不一定会成为their brand loyalty 2) 其他品牌也可以用同样的方法吸引顾客
(A) Many consumers are unlikely to try new brands of products unless offered an inducement to do so.--->irreverent
(B) The consumers whose purchases are strongly influenced by cents-off coupons tend not to become loyal customers of any particular brand.--->correct
(C) Many grocery stores attract customers by doubling the face value of manufacturer's coupons.--->irreverent
(D) Typically less than one-third of the coupons issued by a manufacturer are redeemed by consumers.--->irreverent
(E) A marketing campaign that uses cents-off coupons is most effective when combined with a television advertising campaign.--->irreverent

T: 28s
P: State spokesperson:  Many businesspeople who have not been to our state believe that we have an inadequate road system.  
P: as is obvious from the fact that in each of the past six years, our state has spent more money per mile on road improvements than any other state.
C: Those people are mistaken,
Weaken: spent more money per mile on road than any other state可能是因为本来的公里数就比别人少。
(A) In the spokesperson's state, spending on road improvements has been increasing more slowly over the past six years than it has in several other states.--->irreverent
(B) Adequacy of a state's road system is generally less important to a businessperson considering doing business there than is the availability of qualified employees.--->irreverent
(C) Over the past six years, numerous businesses have left the spokesperson's state, but about as many businesses have moved into the state.--->irreverent
(D) In general, the number of miles of road in a state's road system depends on both the area and the population of the state.--->irreverent
(E) Only states with seriously inadequate road systems need to spend large amounts of money on road improvements.--->correct

作者: echo-LUO    时间: 2018-9-16 11:11
【精练】

premise:People who have specialized knowledge about a scientific or technical issue are systematically excluded from juries for trials where the issue is relevant.
conclusion:Thus, trial by jury is not a fair means of settling disputes involving such issues.

猜想:不能加入陪审团是因为其他原因,而并非not fair
(E) Expert witnesses in specialized fields often command fees that are so high that many people involved in litigation cannot afford their services.
选项:E 因为太贵了...

正确选项:(B) The more a juror knows about a particular scientific or technical issue involved in a trial, the more likely it is that the juror will be prejudiced in favor of one of the litigating parties before the trial begins. 陪审员了解的细节越多,越容易产生偏见
分析原因:expert witnesses和juror无关

【逻辑链】
29. WEAKEN

scientists have been investigaing the way to give advance warning of dangerous earthquakes, and they found the behavior of catfish change before earthquakes. so some investigators proposed catfish can predict dangerous earthquakes.
猜想:可能不仅仅在地震前会
(E) Even very slight, fleeting tremors cause catfish to swim erratically.
选项:E 一点点的震动就会有异动,不能准确预测dangerous earthquakes

30.

the city want to attract new cirizens, thus requiring developers build new houses without cost to the city.
猜想:developers成本提高,使得价格抬高,没人买
(A) Developers would pass along their costs to the buyer, thereby raising the cost of housing units beyond the ability of likely purchasers to afford them.
选项:A 和猜想一致

31.

cents0off coupons is used by manufacturers to get consumers switch to their brand loyaltly, thus the new brand X should use cents-off coupons.
猜想:1. 不一定能吸引;2. 吸引了不一定能留住
(B) The consumers whose purchases are strongly influenced by cents-off coupons tend not to become loyal customers of any particular brand.
选项:B 能够被cent-off coupons吸引的顾客不会是忠诚的顾客

32.
State spokesperson:  Many businesspeople who have not been to our state believe that we have an inadequate road system.  Those people are mistaken, as is obvious from the fact that in each of the past six years, our state has spent more money per mile on road improvements than any other state.
Which of the following, if true, most seriously undermines the reasoning in the spokesperson's argument?
because more monry has been spent on improving road system--->the city has an inadequate road system
猜想:以前太差,投的钱多也没改善完
(E) Only states with seriously inadequate road systems need to spend large amounts of money on road improvements.

选项:E 因为不好所以才要投钱改善

作者: 喜皮    时间: 2018-12-13 13:41
1.—>28s
Premise:people who have technique knowledge are not likely to involve in the trails of jury
Conclusion:thus, trails of jury is not a fair solution to solve the conflicts in this field.
Pre-phrase:[causation error]weaken—>trails of jury is a fair solution/is an accident/another reason why the specific people don’t like involving the trails.

A) The more complicated the issue being litigated,
the less likely it is that a juror without
specialized knowledge of the field involved
will be able to comprehend the testimony
being given.
——>opposite+irrelevant
(B) The more a juror knows about a particular
scientific or technical issue involved in a
trial, the more likely it is that the juror will be
prejudiced in favor of one of the litigating
parties before the trial begins.
——>kept for the first round.the test don’t mention “the more...the more”
(C) Appointing an impartial arbitrator is not a fair
means of settling disputes involving
scientific or technical issues, because
arbitrators tend to favor settlements in which
both parties compromise on the issues.
——>irrelevant
(D) Experts who give testimony on scientific or
technical issues tend to hedge their
conclusions by discussing the possibility of
error.
——>partial support
(E) Expert witnesses in specialized fields often
command fees that are so high that many
people involved in litigation cannot afford
their services.
——>kept for the first round.right
OA:B
忘了结论是【fair solution】,像DE都在讨论别的角度为什么不能让科学家当陪审员。
纠结B的原因是因为B有个程度的递进,感觉像是错的,被打脸了。除此之外,在方向对了,是【他因】也就是不是这个制度“不公平”而是这群人“不公平”
作者: 喜皮    时间: 2018-12-13 13:54
2.——>34s
A) In Japan, which is subject to frequent earthquakes, the behavior of catfish has long been associated with earthquakes.
(B) Mechanical methods for detecting earthquakes have not proved effective.
(C) Tremors lead to the release of hydrogen sulfide gas into water, thereby causing various fish and shellfish to behave erratically.
(D) Careful construction can reduce the dangers posed by earthquakes.
(E) Even very slight, fleeting tremors cause catfish to swim erratically.
E

3.——>45s
A) Developers would pass along their costs to the buyer, thereby raising the cost of housing units beyond the ability of likely purchasers to afford them.
(B) Light, nonpolluting industries have located in the area, offering more jobs and better-paying jobs than do the more-established industries in the area.
(C) Other towns and cities nearby have yet to embark on any comparable plans to attract new citizens.
(D) Most developers see the extra expense of providing municipal facilities as simply one of the many costs of doing business.
(E) Studies show that purchasers of new houses, especially first-time buyers, rank recreational resources as an important factor in deciding to buy a particular house.
A

4.——>45s
A) Many consumers are unlikely to try new brands of products unless offered an inducement to do so.
(B) The consumers whose purchases are strongly influenced by cents-off coupons tend not to become loyal customers of any particular brand.
(C) Many grocery stores attract customers by doubling the face value of manufacturer's coupons.
(D) Typically less than one-third of the coupons issued by a manufacturer are redeemed by consumers.
(E) A marketing campaign that uses cents-off coupons is most effective when combined with a television advertising campaign.
B

5.——>32s
A) In the spokesperson's state, spending on road improvements has been increasing more slowly over the past six years than it has in several other states.
(B) Adequacy of a state's road system is generally less important to a businessperson considering doing business there than is the availability of qualified employees.
(C) Over the past six years, numerous businesses have left the spokesperson's state, but about as many businesses have moved into the state.
(D) In general, the number of miles of road in a state's road system depends on both the area and the population of the state.
(E) Only states with seriously inadequate road systems need to spend large amounts of money on road improvements.
E
作者: 云栈    时间: 2019-10-27 03:43
1.        20s
P: 有科学或者科技知识的人将被系统排除于该问题有关的陪审团外
C:所以trial by jury不是解决相关问题的公平手段
推测:有科学知识背景的人会给陪审团带来不公平因素
选项推测:B
A:越复杂的案件越需要相关专业知识解释,不代表有相关知识的人要参与陪审团,而且相关知识不一定是科学或者科技知识
B:juror知道科学知识会有偏见,所以要把有科学知识背景的人排除
C:只说了仲裁人没说陪审团
D: 无关没说是支持还是反对
E:无关
【这道题推测的时候方向是对的,看答案的时候不知道为什么想反了】

2.21s
P:为了降低地震对生命和财产造成的影响,科学家在调查一些科学方法提前预警地震
P: Catfish在地震来之前会游的很快
C:一些科学家建议用catfish作为预警
推测:catfish只在地震来之前的很短的时间游的很快,不会给人足够时间预防;catfish在其他情况下也会游的很快。
选项分析:E

3.38s
P: 一个城市计划通过新的住房,新的设施例如公园,娱乐场所,图书馆来吸引新的居民。
P: 城市计划的一部分是需要开发商获得许可来建新的住房免费提供这些额外的设施。
推测:对于开发商来说卖房子赚的钱不够建额外设施;开发商把设施的钱加到住房中,房价高,很多人买不起。
选项分析:A

4.41s
P: 工厂给客人发coupon来吸引他们尝试自己的产品,从而让那些顾客对品牌忠诚
C:X是H工厂的主要市场,他们应该发一些coupon,从而吸引大批的潜力客户,变成忠诚客户
推测:发coupon作用不大,或者会带来弊端
选项分析:B

5.        22s
P: 很多没来过我们城市的人觉得我们城市的公路系统不完善。
P: 这个观点是错误的,在过去6年我们州在每英里的公路提升上花的钱比别的州要多
推测:花的钱多但是还是不完善;只有不完善的州才需要花很多钱
选项分析:E

作者: Lincy123    时间: 2020-4-26 00:36
P:涉及到科学技术问题时专业的人能逃脱陪审团的判决
一下子没看懂句子:当涉及技术问题的审判时,有专业背景的人会被排除在外
C:陪审团在涉及这类问题时不是一个公平的方法
A) 问题越复杂,非专业评审越难理解。 增强
B) 陪审越了解科技,就会在审判之前对某一方越有偏见。增强
因为有知识会产生偏见,所以没有知识才更公平。Correct
C) 选用公平的仲裁不是公平手段,因为仲裁倾向于双方妥协的结果。Correct
Arbitrator不好跟jury无关
D)科技专家会通过讨论错误的可能性来规避结论。增强
E) 专家收费太高,一般人要不起。无关

P:catfish在地震前swim erratically
C:用catfish预测地震
Weaken:catfish在其他因素下也会swim erratically
A) 日本catfish的行为一直都和地震相关。增强
B) 机器预测无效。 增强
C) Hydrogen sulfide gas 让catfish swim erratically。 Correct
地震引起的hsg让catfish行为,解释了catfish行为原因,无关
D)小心建设能减少危险。无关
E) 很小的震动都能让catfish se。增强
说明不是地震catfish也会行为奇怪,correct

让获得允许建新房子的开发商提供额外的公共设施
A) 开发商把成本算到业主上,房价很高没人买得起。Correct
B) 工业带来很多工作。无关
C) 其他城市有相同的手段。无关
D)开发商把建造公共设施当作做生意的成本。无关
E) 买房的人把recreational 资源当作第一位。无关

P:发放cents-off优惠券
C:吸引大量潜在顾客变成忠诚顾客
A) 很多消费者如果没有诱因是不会尝试新品牌的。无关
B) 被cents-off优惠券吸引的消费者不会变成任何一个品牌的忠诚顾客。Correct
C) 杂货店用双倍面额优惠券吸引顾客。无关
D)小于三分之一的优惠券被消费者redeemed。无关
E) Cents-off优惠券的营销活动搭配电视广告的时候效果最好。无关

P:过去六年每一年在平均每一公里的道路改善费用都超过其他州
C:没来过我们州的商人认为我们的道路系统不足是错的
Weaken:修理费用多可能是因为路差
A) 每一年修缮费用比上一年减少。无关
B) 商人做生意的考量因素。无关
C) 商人迁入迁出。无关
D)道路的长短与州面积和人口有关。 无关
E) 只有路不好才要花钱修路。Correct






欢迎光临 ChaseDream (https://forum.chasedream.com/) Powered by Discuz! X3.3